Wills and the Administration of Estates MCQS Flashcards

1
Q

A died, survived by his wife (B) and daughter(C). His validly executed will includes only the following gifts:

(a) My house to my wife

(b) £50,000 to my daughter

A’s succession estate comprises the following property (all solely owned):

A house

Bank accounts containing £100,000

Personal chattels

Which of the following correctly explains how A’s estate will be distributed?

A) A is testate because he has a validly executed will. The house will pass to B. C will receive the contents of the bank accounts. The personal chattels will be divided equally between B and C.

B) A is partially intestate because his will does not dispose of his entire estate. The house will pass to B. C will receive £50,000. The remaining £50,000 and A’s personal chattels will be dealt with under the intestacy rules.

C) A is intestate because his will does not dispose of his entire estate. All his property will be dealt with under the intestacy rules.

D) A is intestate because his will does not dispose of his entire estate. The house will pass to B. C will receive £50,000. The remaining £50,000 and A’s personal chattels will be dealt with under the intestacy rules.

E) A is partially intestate because his will does not dispose of his entire estate. The house will pass to B. C will receive the contents of the bank accounts. The personal chattels will be dealt with under the intestacy rules.

A

B) A is partially intestate because his will does not dispose of his entire estate. The house will pass to B. C will receive £50,000. The remaining £50,000 and A’s personal chattels will be dealt with under the intestacy rules.

(Correct: Any property covered by the will is dealt with in accordance with that will. The remainder passes in accordance with the intestacy rules)

How well did you know this?
1
Not at all
2
3
4
5
Perfectly
2
Q

Which of the following would be included in a deceased person’s succession estate?

A) A vested remainder interest in a trust, a share of a house owned as tenants in common and cash held in a bank account.

B) A life interest in a trust, a share of a house owned as tenants in common and cash held in a bank account.

C) A remainder interest in a will trust which is contingent on the deceased surviving the life tenant, a share of a house owned as tenants in common and a life assurance policy not written in trust.

D) A remainder interest in a trust which is contingent on the deceased surviving the life tenant, a share of a house owned as tenants in common and a life assurance policy written in trust.

E) A vested remainder interest in a trust, a share of a house owned as joint tenants and cash held in a bank account.

A

A) A vested remainder interest in a trust, a share of a house owned as tenants in common and cash held in a bank account.

(Correct: All of these assets would be included in the succession estate. Note that the position would be different if the deceased held a life interest in the will trust or the house was held as joint tenants)

How well did you know this?
1
Not at all
2
3
4
5
Perfectly
3
Q

A died yesterday. His estate consisted of the following assets (all solely owned unless otherwise stated:

House £400,000 Antiques worth £180,000 (owned with B as joint tenants). Bank accounts £25,000 Life Assurance Policy £50,000 (not written in trust)

A was also the life tenant of C’s estate. There was £100,000 in the trust fund at A’s death. B is the remainderman.

What is the value of A’s succession estate?

A) £655,000

B) £565,000

C) £575,000

D) £475,000

D) £425,000

A

D) £475,000

(Correct. A was the sole owner of the house, the bank accounts and the life policy proceeds (which were not written in trust). The antiques pass to B via survivorship)

How well did you know this?
1
Not at all
2
3
4
5
Perfectly
4
Q

The following extract from official copies for a freehold property in London contains the entire Proprietorship Register for that property. Please read the extract and the information that follows and then answer the question below.

Proprietorship Register

Title Absolute

Proprietor(s): LUCY WATSON AND OWAIN HUTCHISON: of 1 Greenslade Avenue, London (W12 7JH).
The value as at 7 October 2001 was £375,000.
RESTRICTION: Except under an order of the Registrar no disposition by a proprietor of the land is to be registered without the consent of the proprietor of the charge dated 7 October 2001 in favour of Rydale Bank referred to in the Charges Register.
The heads of terms for the sale of the above property state that Lucy Watson is the seller. Owain Hutchison has died.

Which ONE of the following statements is the most accurate advice to a buyer client on the issue of who will sell the property?

A) Lucy Watson can sell on her own as she and Owain Hutchison held the property as beneficial joint tenants and on Owain’s death his beneficial interest passed to Lucy under the doctrine of survivorship so the buyer just needs to see an official copy of Owain’s death certificate.

B) Lucy Watson and Rydale Bank will sell the property together as Owain’s interest in the property passed to the Bank, who have a mortgage over the property, on his death.

C) The Restriction on the Proprietorship Register indicates that the buyer must purchase the property from Rydale Bank.

D) Lucy Watson and Owain Hutchison held the property as tenants in common so the buyer would need to see an official copy of Owain Hutchison’s death certificate and a second trustee should be appointed to receive the purchase monies with Lucy Watson thereby overreaching any beneficial interests in the property. Both Lucy and the second trustee will be the sellers.

A

A) Lucy Watson can sell on her own as she and Owain Hutchison held the property as beneficial joint tenants and on Owain’s death his beneficial interest passed to Lucy under the doctrine of survivorship so the buyer just needs to see an official copy of Owain’s death certificate

(Correct. Lucy and Owain were beneficial joint tenants (because there was no tenant in common restriction in the Proprietorship Register) so on Owain’s death, Lucy could sell on her own because Owain’s beneficial interest had passed to Lucy under the doctrine of survivorship. The buyer just needs to see an official copy of Owain’s death certificate)

How well did you know this?
1
Not at all
2
3
4
5
Perfectly
5
Q

Which one of these entries in the Proprietorship Register relates to co-owners of the property?

A) The Transfer to the Proprietors contains a covenant to observe and perform the covenants referred to in the Charges Register and of indemnity in respect thereof.

B) RESTRICTION: No disposition by a sole proprietor of the registered estate (except a trust corporation) under which capital money arises is to be registered unless authorised by an order of the court.

C) RESTRICTION: no disposition of the registered estate by the proprietor of the registered estate is to be registered without the consent signed by the proprietor for the time being of the charge dated the 17 June 2005 in favour of Redminister Building Society referred to in the charges register.

A

B) RESTRICTION: No disposition by a sole proprietor of the registered estate (except a trust corporation) under which capital money arises is to be registered unless authorised by an order of the court

(Correct. This is the tenant in common Restriction)

How well did you know this?
1
Not at all
2
3
4
5
Perfectly
6
Q

If a buyer pays the purchase money to two trustees, why does the buyer not need to worry about the beneficial interests of any other deceased co-owners?

A) Because any such interests will be overreached.

B) Because the rule of survivorship applies.

C) Because only two people can hold the legal estate.

D) Because the buyer only needs to worry about the legal estate, not any beneficial interests.

A

A) Because any such interests will be overreached

(Correct. You have understood the concept of overreaching)

How well did you know this?
1
Not at all
2
3
4
5
Perfectly
7
Q

A man died intestate a month ago. He had never been married or in a civil partnership and had never had any children. Both of the man’s parents died before him. The man was survived by his brother (aged 20), his sister (aged 16, and pregnant with her first child when her brother died), and his aunt (his mother’s sister). The man’s sister gave birth to a baby boy (the man’s nephew) 6 months after the man died. The man’s sister died during childbirth, aged 17.

Who is entitled to share in the distribution of the man’s estate?

A) His aunt only.

B) His brother and his nephew only.

C) His brother only.

D) His aunt, his brother and his nephew only.

E) His brother and his sister’s estate only

A

B) His brother and his nephew only

(Correct. After spouse, issue and parents the next category of relative entitled to inherit are the siblings of the intestate. The siblings inherit on the terms of the statutory trust so in this case, the brother and sister are entitled to half of the estate each on the terms of the statutory trust i.e. contingent on reaching the age of 18.
The sister died after the intestate but before reaching the contingent age. This means her share never vested and is therefore not part of her estate. Any option suggesting the sister’s estate would benefit is therefore incorrect.
However, because the sister died leaving her own issue, the substitution limb of the statutory trust would apply. The share that would have passed to the man’s sister, passes instead to her children who are alive or en ventre sa mere (a French expression meaning “in its mother’s belly” which refers to a child conceived but not yet born) at the date of the intestate’s death, contingent upon their attaining the age of 18 or marrying earlier. Although the man’s nephew was not born until after his death, his sister was pregnant at this time and therefore the nephew satisfies the criteria assuming he reaches 18.
The man’s aunt would only inherit if the man had no siblings, or nieces /nephews)

How well did you know this?
1
Not at all
2
3
4
5
Perfectly
8
Q

A woman died intestate a month ago. At the time of her death she was separated from her husband of 10 years. The woman and her husband had discussed getting divorced but no formal proceedings had begun. When she died, the woman was living with her son (aged 17 years) and her step-son (aged 18 years). The woman’s daughter (aged 25) lives with her husband and their new baby, the woman’s only grandchild.

Who is entitled to share in the distribution of the woman’s estate?

A) The husband, son, daughter and grandchild only.

B) The husband, son, step-son and daughter only.

C) The son and daughter only.

D) The husband, son and daughter only.

E) The son and step-son only.

A

D) The husband, son and daughter only

(Correct. Where a person dies intestate and is survived by a spouse and issue, only those relatives will take a share of the deceased’s estate. Here, the woman was survived by her husband (it does not matter that they were not living together), and her son (it does not matter how old her son is) and daughter (it makes no difference whether she is living with the deceased or not). The step-son has no entitlement to the estate. Nor does the grandchild if the parent is alive at the date of the intestate’s death)

How well did you know this?
1
Not at all
2
3
4
5
Perfectly
9
Q

A man died intestate a month ago. His estate comprised a property owned as joint tenants with his brother, a bank account in his sole name, personal possessions, cash in the house, and a discretionary lump sum payable under the terms of the man’s pension scheme (nominated in favour of the man’s brother).

Which of the following lists the assets that will be distributed in accordance with the intestacy rules?

A) The property, bank account and personal possessions only.

B) The property, bank account, personal possessions, cash, and the amount payable under the discretionary pension scheme.

C) Bank account, personal possessions and cash only.

D) Bank account, personal possessions, cash, and the amount payable under the discretionary pension scheme only.

E) The property, bank account, personal possessions and cash only

A

C) Bank account, personal possessions and cash only

(Correct. Items owned as joint tenants will pass in accordance with the rules of survivorship. Therefore, his property will pass directly to his brother and not under the intestacy rules. The discretionary pension lump sum is payable in accordance with the nomination that was made, so will pass directly to his brother and not under the intestacy rules. The remainder of the estate assets are included in the deceased’s succession estate)

How well did you know this?
1
Not at all
2
3
4
5
Perfectly
10
Q

A man died intestate a month ago. He was survived by his civil partner and their son (aged 21). The man’s estate comprises a ½ share of a property owned as tenants in common with his civil partner (value of whole £500,000), a savings account (£40,000), a vehicle used solely for business purposes (£5,000) and personal possessions (£20,000).

Which of the following best describes what the man’s civil partner and son are entitled to under the intestacy rules?

A) The man’s civil partner will receive chattels (worth £25,000), a statutory legacy of £322,000, and half of the remainder. The man’s son will receive a contingent interest in the other half of the remainder.

B) The man’s civil partner will receive the entire estate passing under the intestacy rules. The man’s son will receive nothing.

C) The man’s civil partner will receive chattels (worth £25,000), a statutory legacy of £322,000, and half of the remainder absolutely. The man’s son is entitled to a vested interest in the other half of the remainder.

D) The man’s civil partner will receive chattels (worth £20,000), a statutory legacy of £322,000, and half of the remainder. The man’s civil partner is entitled to have the property appropriated to them in addition to her entitlement under intestacy. The man’s son will receive a contingent interest in the other half of the remainder.

E) The man’s civil partner will receive chattels (worth £20,000), a statutory legacy of £322,000, and half of the remainder. The man’s civil partner is entitled to have the property appropriated to them as part of their entitlement under the intestacy. The man’s son will receive a vested interest in the other half of the remainder.

A

E) The man’s civil partner will receive chattels (worth £20,000), a statutory legacy of £322,000, and half of the remainder. The man’s civil partner is entitled to have the property appropriated to them as part of their entitlement under the intestacy. The man’s son will receive a vested interest in the other half of the remainder.

(Correct. The civil partner is entitled to receive chattels worth £20,000 (the personal possession but not the business assets), plus a statutory legacy of £322,000, and half of the remainder. As the half-share of the property falls within the estate passing under intestacy, the civil partner can request that the property is transferred to them as part of their entitlement (not in addition to). The son will receive the other half of the remainder on statutory trusts. As he is over the age of 18 his share will be vested)

How well did you know this?
1
Not at all
2
3
4
5
Perfectly
11
Q

A woman died intestate a month ago. She had never been married or in a civil partnership. The woman was living with her only grandchild (aged 1). The woman’s son, the grandchild’s father, died before the woman.

The woman’s brother (her only sibling), both of her parents, and her daughter-in-law (the wife of her son) are still alive.

Who is entitled to share in the distribution of the woman’s estate?

A) Her daughter-in-law only.

B) Her daughter-in-law and grandchild only.

C) Her grandchild only.

D) Her granddaughter and brother only.

E) Her parents only.

A

C) Her grandchild only

(Correct. Where a person dies intestate and is survived by issue but no spouse/civil partner, the issue will inherit the whole of the estate. If a child of the intestate dies before their parent, but leaving their own issue, the intestate’s grandchild will inherit the share of the estate their parent would have inherited.
The other options were incorrect because:
-the parents and siblings of an intestate will only inherit if the intestate is not survived by either spouse or issue.
-Where a person due to inherit dies before the intestate’s (here her son) then the son’s spouse would not be entitled to any share of the intestate’s estate)

How well did you know this?
1
Not at all
2
3
4
5
Perfectly
12
Q

A woman executes her will in accordance with the requirements of s.9 Wills Act 1837. There is no doubt that she has testamentary capacity. The will gives the whole of the woman’s estate to her son and nothing to her daughter.

The daughter tells you that her brother encouraged their mother to make a will leaving everything to him and nothing to the daughter. The son confirms that he did ask his mother to make a will leaving everything to him because his sister is “not to be trusted with money”.

The daughter tells you that her mother would never have agreed or intended to give everything to her son.

Is the woman’s will valid?

A) No. The son encouraged the testator to make a will that favoured him over his sister and therefore the will does not reflect her true intention.

B) Yes, unless the daughter can provide evidence that her brother placed her mother under so much pressure that she only signed the will because she felt forced to and then when she signed it the woman did not actually want her son to inherit the whole of her estate.

C) Yes. The woman had testamentary capacity and executed the will properly. This raises a presumption that she had the requisite intention to make this will.

D) No, unless the son can provide evidence that his mother intended for him to inherit the whole of her estate.

E) Yes, provided that the son can produce evidence that his sister is reckless with money.

A

B) Yes, unless the daughter can provide evidence that her brother placed her mother under so much pressure that she only signed the will because she felt forced to and then when she signed it the woman did not actually want her son to inherit the whole of her estate

(The will has met the execution requirements and the testator has testamentary capacity. For the will to be invalid, it must therefore be shown that the son had unduly influenced his mother to the extent that she signed a will she did not intend to make. It is not unlawful for the brother to have persuaded his mother to prefer him over his sister. The burden of proof is on the sister to provide evidence that undue influence occurred, not for the brother to provide evidence that intention was present)

How well did you know this?
1
Not at all
2
3
4
5
Perfectly
13
Q

*/A woman died intestate two months ago and her estate was distributed between her three adult children. The woman had never married or entered a civil partnership. On the date of her death she had been living with her long-term partner and his son (aged 15) for the previous 10 months. The woman provided financial maintenance for both her partner and her step-son throughout the period they all lived together.**

Who is entitled to bring a claim against the woman’s estate under the Inheritance (Provision for Family and Dependants) Act 1975?

A) The woman’s step-child only.

B) The woman’s children and step-child.

C) The woman’s partner, children and step-child.

D) The woman’s partner only.

E) No one is entitled to bring a claim because the woman did not leave a will.

A

C) The woman’s partner, children and step-child

(Correct. The woman’s partner could apply as a person maintained at the date of death under s.1(1)(e). However, he could not apply as a cohabitee under s.1(1A), as they were not living together for the two years immediately preceding her death.
The woman’s step-child could possibly apply as a person treated as a child of the deceased under s.1(1)(d), or if the facts did not support this, then as a person being maintained under s.1(1)(e).
The woman’s children could apply under s.1(1)(c). That they are adult children and have received her estate under intestacy does not preclude a claim but would affect the likelihood of them receiving any award)

How well did you know this?
1
Not at all
2
3
4
5
Perfectly
14
Q

A new client aged 40 wishes to give instructions to make a will.

Which of the following most accurately describes best practice when taking the instructions?

A) Ask questions to establish if the client meets the requirements of the test in Banks v Goodfellow.

B) Ask questions to establish if the client meets the requirements of the test in Parker v Felgate.

C) Contact the client’s doctor for an opinion on whether the client has testamentary capacity to make a will.

D) Ask questions to establish if the client meets the statutory test for capacity under the Mental Capacity Act 2005.

E) Ask the client if they have testamentary capacity to make a will.

A

A) Ask questions to establish if the client meets the requirements of the test in Banks v Goodfellow.

(This is best practice in a case where your client is young and healthy)

How well did you know this?
1
Not at all
2
3
4
5
Perfectly
15
Q

A man died 8 months ago and the grant of probate was issued four months later. By his will the man left all of his estate to his son and nothing to his partner with whom he had been living for the last 3 years. The man’s partner wishes to bring a claim under the Inheritance (Provision for Family and Dependants) Act 1975 (the ‘Act’).

Is the man’s partner able to bring a claim under the Act?

A) No - because it has been less than 6 months since the date of the grant of probate.

B) No - because the man died more than 6 months ago.

C) Yes

D) No - because they were not married to or in a civil partnership with the man.

E) Only if a court order granting permission for the deadline to be extended is obtained.

A

C) Yes

(Correct. The deadline for bringing a claim (without court permission) is 6 months from the date of the grant of representation. A claim can be brought under the Act before the grant is issued)

How well did you know this?
1
Not at all
2
3
4
5
Perfectly
16
Q

A woman died interstate two months ago and her estate was distributed between her three adult children. The woman had never marred or entered a civil partnership. On the date of her death she had been living with her long-term partner and his son (aged 15) for the previous 10 months. The woman provided financial maintenance for both her partner and her step-son throughout the period they all lived together.

Who is entitled to bring a claim against the woman’s estate under the Inheritance (Provision for Family and Dependants) Act 1975?

A) The woman’s partner, children and step-child.

B) The woman’s children and step-child.

C) No one is entitled to bring a claim because the woman did not leave a will.

D) The woman’s step-child only.

E) The woman’s partner only.

A

A) The woman’s partner, children and step-child

(Correct. The woman’s partner could apply as a person maintained at the date of death under s.1(1)(e). However, he could not apply as a cohabitee under s.1(1A), as they were not living together for the two years immediately preceding her death.
The woman’s step-child could possibly apply as a person treated as a child of the deceased under s.1(1)(d), or if the facts did not support this, then as a person being maintained under s.1(1)(e).
The woman’s children could apply under s.1(1)(c). That they are adult children and have received her estate under intestacy does not preclude a claim but would affect the likelihood of them receiving any award)

How well did you know this?
1
Not at all
2
3
4
5
Perfectly
17
Q

A man died 8 months ago and the grant of probate was issued four months later. By he will the man left all of his estate to his son and nothing to his partner with whom he had been living for the last 3 years. The man’s partner wishes to bring a claim under the Inheritance (Provision for Family and Dependants) Act 1975 (the ‘Act’).

Is the man’s partner able to bring a claim under the Act?

A) Yes

B) Only if a court order granting permission for the deadline to be extended is obtained.

C) No - because it has been less than 6 months since the date of the grant of probate.

D) No - because the man died more than 6 months ago.

E) No - because they were not married to or in a civil partnership with the man.

A

A) Yes

(Correct. The deadline for bringing a claim (without court permission) is 6 months from the date of the grant of representation. A claim can be brought under the Act before the grant is issued)

How well did you know this?
1
Not at all
2
3
4
5
Perfectly
18
Q

If you are acting for a client who lacks testamentary capacity, should you take their instructions for preparing a will?

A) No

B) Yes

A

A) No

(If a client lacks capacity they cannot make a valid will and a solicitor should not accept their instructions to prepare one)

How well did you know this?
1
Not at all
2
3
4
5
Perfectly
19
Q

A client gives you instructions to prepare a new will which leaves the whole of their estate to their civil partner. You consider the test in Banks v Goodfellow and are certain that your client has testamentary capacity.

Five days after giving the instructions the client was involved in a car accident which left them disorientated and confused. You visit the client in hospital and bring with you the will you have prepared. The client remembers meeting you to give instructions for a will and is happy to sign the will you have prepared in accordance with those instructions. However, when talking with the client you have some doubt whether they still satisfy the test for capacity and their memory of your meeting and the points discussed seems a little unclear.

Which one of the following is correct?

A) It is possible for the client to make a valid will in the hospital even if they lack capacity when it is executed because they had capacity when giving instructions and the terms of the will are rational.

B) The client could execute a valid will in the hospital but only if they satisfy the test for capacity in the Mental Capacity Act 2005.

C) The client must have capacity at the time of execution for the will to be valid.

D) It is possible for the client to make a valid will in the hospital even if they lack capacity when it is executed because they had capacity when giving instructions and understand they are now signing a will for which they had previously given instruction.

E) The client could make a valid will in the hospital provided a doctor acts as a witness.

A

D) It is possible for the client to make a valid will in the hospital even if they lack capacity when it is executed because they had capacity when giving instructions and understand they are now signing a will for which they had previously given instruction.

(Correct. The rule in Parker v Felgate is an exception to the requirement that the client has capacity at point of execution. Provided they had capacity when giving instructions, the will was prepared in accordance with those instructions and the testator understands they are signing a will for which they had previously given instructions they don’t need to satisfy the Banks v Goodfellow test at the point of signing.
The other answers were incorrect because:
- The test in the Mental Capacity Act 2005 is not the test used for testamentary capacity
- There is no requirement for a doctor to witness a will
- Whether or not the will is rational is not a factor to take into account when applying the rule in Parker v Felgate)

How well did you know this?
1
Not at all
2
3
4
5
Perfectly
20
Q

You have been instructed by a client whose elderly mother is in a nursing home and suffering from dementia. Your client’s mother currently lacks capacity which she will not regain.

Your client’s mother is a widow. Her current will divides her estate equally between her three children, including your client. The will was duly executed by the client’s mother and witnessed by two of her friends.

Your client is concerned that their mother may have lacked capacity when she made her will six months ago as your client was expecting to receive more than a 1/3 share of the estate.

Which one of the following most accurately describes the legal position should your client wish to challenge the validity of the will?

A) There is always a presumption in favour of capacity and therefore your client must prove their mother did not have capacity when she made her will.

B) If your client’s mother did not have capacity at the time she made the will it is invalid. As she will not regain capacity it is no longer possible for her to make a new will.

C) There will be a presumption that your client’s mother had capacity when she made her will unless your client has evidence to suggest otherwise.

D) There is a presumption that your client’s mother lacked capacity when she made her will because she is elderly (unless the ‘golden rule’ was followed).

E) There is a presumption that your client’s mother lacked capacity when she made her will because of her current medical condition.

A

C) There will be a presumption that your client’s mother had capacity when she made her will unless your client has evidence to suggest otherwise

(Where the will on the face of it does not raise any concerns, because its terms are rational and it has been properly signed, then capacity is presumed. However, if evidence that raises doubt is provided however the presumption is rebutted and the burden of proof reverts to the propounder of the will to demonstrate the testator satisfied the Banks v Goodfellow test.
The other options were incorrect because:
- If a testator lacks capacity it may be possible for a statutory will to be made for them
- The age of the testator, and whether or not the golden-rule was followed, do not affect the general rule regarding presumption in favour of capacity where the will is rational and has been properly executed
- It is not correct that the person challenging a will must prove the lack of capacity
- The current medical condition is not a determining factor- the testator’s health at the time of execution is)

How well did you know this?
1
Not at all
2
3
4
5
Perfectly
21
Q

A will written in English contains the following attestation clause:

“Signed by the above named Testator in our joint presence and then by us in his after this document had been read to the Testator when the testator seemed thoroughly to understand and approve the contents”

In which circumstances should this form of attestation be used?

A) Where a testator is blind.

B) Where a testator is blind or is not signing the will on their own behalf.

C) Where a testator is blind or illiterate.

D) Where suspicious circumstances surround the execution of the will.

E) Where a testator is blind, illiterate or does not understand English.

A

C) Where a testator is blind or illiterate

(A blind or illiterate testator cannot read the will and therefore to raise a presumption that knowledge and approval was present it needs to be clear that the will was read out loud and that the testator understood what was said before it was signed. The other options were incorrect because: - A testator who does not understand English is not aided by the will being read out loud unless it is clear it was read in the testator’s preferred language by someone who was able to translate it - A testator who does not sign the will himself doesn’t necessarily need it to be read out loud, but the attestation clause should refer to the fact that he did not sign - Where there are suspicious circumstances this form of attestation is not helpful)

How well did you know this?
1
Not at all
2
3
4
5
Perfectly
22
Q

A testator die recently. Three weeks before he died, the testator made a new will leaving all of his assets, apart from a small cash gift to a local charity, to the children of his current marriage. The testator’s previous will made provision for his child from an earlier marriage, but this child does not benefit under the current will.

You have been instructed by the child of the testator from the earlier marriage. Your client spoke to the testator a few days before he died. The testator explained that his wife “kept on at him all the time” to make a will in favour of their children only, and to keep her quiet he signed a new will that reflected what she wanted.

The testator had capacity and knew and approved of the terms of the new will when it was duly executed.

Which of the following is correct regarding a claim of undue influence?

A) A claim of undue influence is not possible because the testator had both capacity and knowledge and approval when they executed their will.

B) If a claim of undue influence succeeds the will would be invalid and the local charity cannot receive the cash gift.

C) A claim of undue influence will succeed if your client provides evidence that the testator’s spouse persuaded the testator to make the will which excluded your client.

D) A claim of undue influence will succeed if your client provides evidence that when the testator signed the will his true intention was not reflected in its terms.

E) A claim of undue influence is not possible because the testator’s spouse does not benefit under the new will.

A

D) A claim of undue influence will succeed if your client provides evidence that when the testator signed the will his true intention was not reflected in its terms

(Even where a testator has capacity and knowledge and approval, if the will was made as a result of undue influence it will not be valid because it does not reflect the testator’s true intention. The other options were incorrect because: - it is not unlawful to encourage someone to make a will or persuade them that certain provisions should be included. Undue influence occurs where a testator is coerced into making a will that is contrary to their true intention. - It is not a requirement that the person committing the wrong unduly influences the testator to make direct provision for themselves - A testator can know and approve of their will but still be acting as a result of undue influence - Where a testator has been unduly influenced in respect of a particular gift (here the gift to the children) it is still possible for the remainder of the will to take effect)

How well did you know this?
1
Not at all
2
3
4
5
Perfectly
23
Q

You are reviewing two wills.

When the first will was signed the testator told the witnesses “I’ve not bothered to read anything the lawyer has sent me – it’s all a bit complicated to be honest, but I’m sure they’ve done the right thing”.

When the second will was executed the testator had hurt their hand and arranged for someone else to sign the will on their behalf.

Both wills were drafted by solicitors and duly executed with a standard form attestation clause. Both testators had testamentary capacity.

Assuming there is no other relevant information, which one of the following is correct with regards knowledge and approval of the wills?

A) Affidavit evidence that knowledge and approval were present at execution is required in respect of both wills.

B) Knowledge and approval will be presumed in respect of both wills because the testator had capacity when the wills were made.

C) Knowledge and approval will be presumed in respect of both wills because the testator had capacity when the wills were made and they were duly executed.

D) The first will is invalid due to lack of knowledge and approval. Affidavit evidence that knowledge and approval were present at execution is required in respect of the second will.

E) The first will is invalid due to lack of knowledge and approval. The second will is invalid because the attestation clause was not updated to make it clear the testator did not sign the will themselves.

A

D) The first will is invalid due to lack of knowledge and approval. Affidavit evidence that knowledge and approval were present at execution is required in respect of the second will

(A testator must intend to make the particular will they sign. This means they should know and approve of its contents and the contents of the will must have been read and understood. The first testator has not read or understood his will so did not have knowledge of its contents when it was signed.
The second will was signed by someone else on behalf of the testator. This means knowledge and approval will not be presumed. As the attestation clause was not amended to reflect this, affidavit evidence to prove knowledge and approval will be required. The other options were incorrect because: - facts suggest the first testator lacked knowledge and approval – so affidavit evidence will not help. - although testamentary capacity and due execution usually lead to a presumption of knowledge and approval, this does not apply where someone signs on behalf of a testator - an inadequate attestation clause does not render a will invalid)

How well did you know this?
1
Not at all
2
3
4
5
Perfectly
24
Q

”Signed by the above named [TESTATOR] in our joint presence“

Which one of the following most accurately summarises the effect of a will containing this clause?

A) If a will included this attestation clause proof of the testator’s intention would be required when seeking to admit the will to probate following the testator’s death.

B) If a will included this attestation clause the will would not be valid.

C) If a will included this attestation clause there would be a presumption that the will was executed in accordance with the requirements of s 9 Wills Act 1837.

D) If a will included this attestation clause it would be incomplete because there is no reference to the will having been read out loud to the testator.

E) If a will included this attestation clause, proof of due execution would be required when seeking to admit the will to probate following the testator’s death.

A

E) If a will included this attestation clause, proof of due execution would be required when seeking to admit the will to probate following the testator’s death.

(Correct. A properly drafted attestation clause raises a presumption that the will was executed in accordance with the requirements of s 9 WA (a presumption of due execution). Therefore it should refer to the testator and the witnesses signing in the presence of each other. This clause makes no reference to the witnesses signing in the presence of the testator so is incomplete. However, there is no legal obligation to include an attestation clause, nor is any specific form of attestation required, so a poorly drafted clause does not invalidate the will. However, proof of due execution is required, usually an affidavit of due execution sworn by the witnesses.
The other answers were incorrect because:
- A missing or poorly drafted attestation clause does not invalidate the will
- A properly drafted attestation clause does not give rise to a presumption of the testator’s intent
- The attestation clause would only mention the will have having been read out loud to a testator where the testator unable to read the will for himself)

How well did you know this?
1
Not at all
2
3
4
5
Perfectly
25
Q

A testator died recently. The testator’s will contains a gift of £10,000 to the testator’s brother and gives the remainder of the estate equally to the testator’s spouse and children. The testator’s spouse, and the spouse of the testator’s brother, witnessed the will.

Which one of the following accurately describes the effect of the man’s will?

A) The will has no effect as it was not properly executed.

B) The man’s brother and children will inherit under the will.

C) The man’s spouse, brother and children will inherit under his will.

D) The man’s spouse and brother will inherit under his will.

E) Only the man’s children will inherit under his will.

A

E) Only the man’s children will inherit under his will.

(Correct. Where a beneficiary (or their spouse) witnesses a will the beneficiary cannot inherit under the will. In this example the testator’s spouse and the spouse of the testator’s brother witnessed the will so neither the spouse nor brother will receive any benefit. Only the gift to the children is effective.
The other options were incorrect because:
S.15 Wills Act only applies to gifts under the will – it does not affect whether or not the will was properly executed
S.15 applies to a gift to a beneficiary who witnesses a will and any gift to their spous)

How well did you know this?
1
Not at all
2
3
4
5
Perfectly
26
Q

Your client arranges to sign their will at home and invites a married couple who live next door to act as witnesses. The three of them gather in the same room and the testator produces the will ready for signing. One of the witnesses leaves the room to take a phone call. Before they return, the testator signs the will in the presence of the other witness, who then signs the will in the presence of the testator. When the first witness returns to the room, the testator acknowledges their signature and the first witness then signs the will in the testator’s presence.

Has the will been executed in accordance with the formal requirements of s.9 Wills Act 1837?

A) The will has been properly executed.

B) The will has not been properly executed because one of the witnesses did not see the testator sign the will.

C) The will has not been properly executed because the testator and both witnesses must all be in the same room throughout the execution process.

D) The will has not been executed properly because the witnesses signed the will in the presence of the testator but not each other.

E) The will has not been properly executed because the witnesses are married to each other.

A

A) The will has been properly executed

(Correct. S 9 Wills Act 1837 requires the testator to sign the will (or acknowledge his signature) in the presence of two witnesses.
The other options were incorrect because:
- S 9 permits the testator to acknowledge his signature
- The marital status of the witnesses is irrelevant
- Provided s. 9 requirements are met there is no additional obligation for everyone in involved to be present throughout the execution process (although this is good practice)
- There is no requirement for the witnesses to sign the will in the presence of each other, only the testator)

How well did you know this?
1
Not at all
2
3
4
5
Perfectly
27
Q

Review the following extracts from a valid will:

Clause 1: I hereby REVOKE all former wills and testamentary dispositions and declare this to be my last will

Clause 2: I APPOINT ANDREW BATES of […] and CATHY DENNIS of […] (hereinafter called “my Trustees” which expression where the context admits includes any trustee hereof for the time being) to be the executors and trustees of this my will but if either or both of them shall die in my lifetime or be unable or unwilling to act as my executor I appoint ESTHER FISHER of […] to fill any vacancy thereby arising

The testator died last week. Andrew died before the testator. You may assume anyone entitled to act as executor or trustee is willing and able to do so.

Which one of the following is correct?

A) Cathy would act as the sole executor of the estate. If any trust arises following the testator’s death Esther would need to act as trustee with Cathy so there are two trustees.

B) Cathy and Esther will be appointed as executors of the estate. If any trust arises following the testator’s death, Cathy and Esther will also be the trustees.

C) Cathy and Esther will be appointed as executors of the estate. However, if any trust arises following the testator’s death, it is not clear who will act as the trustees.

D) Clause 1 needed to include the date the will was signed to avoid any confusion about which is the last valid will the testator made.

E) Clause 1 did not revoke any codicils made before this will.

A

B) Cathy and Esther will be appointed as executors of the estate. If any trust arises following the testator’s death, Cathy and Esther will also be the trustees

(Correct. Andrew cannot act as he has pre-deceased the testator and Clause 2 appoints Esther in his place. Clause 2 also confirms that the executors should also act as trustees of any trust.
The other options were incorrect because:
- The revocation clause revokes any will or codicil made previously
-Esther is appointed as a replacement for Andrew (there is no option for Cathy to act alone)
-A will should contain a date but this can be at the end or as part of the commencement clause – it does not need to be part of the revocation clause)

How well did you know this?
1
Not at all
2
3
4
5
Perfectly
28
Q

Your client is a lay executor appointed under a valid will that contains a number of express administrative powers for executors, however, no express charging provisions are included.

Which one of the following is correct?

A) Your client can charge for their time but only in accordance with the statutory power.

B) If there is a conflict between any of the express powers in the will and the equivalent statutory power your client may act in accordance with whichever confers the widest power.

C) Your client cannot charge for their time as there is no express power allowing this.

D) Your client can charge for their time as there is no express power preventing this.

E) If there is a conflict between any of the express powers in the will and the equivalent statutory power, your client must act in accordance with the statutory power

A

C) Your client cannot charge for their time as there is no express power allowing this.

(Correct. There is no express power to charge in the will and the statutory power to charge applies only to professionals. The other options were incorrect because:
Your client cannot charge for their time
Express powers take priority over the implied statutory terms in a will where there is conflict)

How well did you know this?
1
Not at all
2
3
4
5
Perfectly
29
Q

Review the following extracts from a valid will:

Clause 2: I WISH for any part or parts of my body where possible to be used for transplantation and for the treatment of others

Clause 4: I APPOINT my brother and sister jointly to be the guardians of any of my children who have not attained the age of 18 at the death of the survivor of myself and my civil partner

The testator died last week and is survived by her civil partner and their first daughter (aged 25), second daughter (aged 17) and their son (aged 14).

Which one of the following is correct?

A) No guardians are appointed following the testator’s death.

B) The testator’s brother and sister will be appointed as guardians for the testator’s second daughter and son only.

C) The testator’s brother and sister will be appointed as guardians for the three children.

D) Either the testator’s brother or the testator’s sister will be appointed as sole guardian of the testator’s second daughter and son.

E) The executors must ensure the testator’s body is used for transplantation and for the medical treatment of others.

A

A) No guardians are appointed following the testator’s death

(Correct. As the testator is survived by their civil partner no guardians are appointed by clause 4. The other options were incorrect because no guardians are appointed as the testator’s civil partner is still alive, and Clause 2 imposes a moral obligation rather than a legal one)

How well did you know this?
1
Not at all
2
3
4
5
Perfectly
30
Q

“ I give my gold diamond engagement ring absolutely and free of tax and costs of transfer to my niece”

You have just discovered that the testator sold the gold diamond engagement ring for £4,000 a month ago. However, there is still a platinum engagement ring among the testator’s personal possessions.

What will the testator’s niece receive?

A) £4,000

B) A gold engagement ring that the personal representatives must buy as a replacement.

C) Nothing

D) The testator’s niece may choose either £4,000 or the platinum ring but only if it has not been left to another beneficiary.

E) Platinum engagement ring but only if it has not been left to another beneficiary.

A

C) Nothing

(Correct. The gift of the ring is a specific legacy. If the testator no longer owns the item at the date of their death then the gift in the will cannot be given effect to. In the absence of any express clause providing for an alternative gift, the clause fails and the beneficiary will receive nothing.
The other options were incorrect because:
· The original gift adeems because the testator does not own the item specifically left by the will
-There is no cash or chattel alternative which can be given as there are no express words permitting this)

How well did you know this?
1
Not at all
2
3
4
5
Perfectly
31
Q

A testator owns two properties. A residential house and a holiday home that is let out.

The testator owns the residential house as joint tenants with their civil partner and owns the holiday home as tenants in common with their sister. The testator wants their civil partner to inherit the residential house and their sister to inherit the holiday home.

The rest of the estate (the residue) will be left to the testator’s children.

When you draft the testator’s will which of the following are required?

A) A clause giving the testator’s share of the residential house to their civil partner and another clause giving the testator’s share of the holiday home to their sister.

B) No express clauses are required to achieve the testator’s wishes.

C) A clause giving the testator’s share of the holiday home to their sister.

D) A clause severing the joint tenancy of the residential house.

E) A clause giving the testator’s share of the residential house to their civil partner.

A

C) A clause giving the testator’s share of the holiday home to their sister

(Correct. The holiday home is owned as tenants in common and therefore the testator’s share passes in accordance with the will and is not automatically inherited by the sister as the other co-owner. As we know the residue will not be left to the testator’s sister, an express clause giving the testator’s share of the holiday home to their sister is required. The residence owned as joint tenants will pass by survivorship to the surviving co-owner automatically. As the testator wants their civil partner to inherit in any event no express clause in the will is required to achieve the testator’s wishes.
The other options were incorrect because: - No severance of the joint tenancy is required as the testator wants the surviving co-owner to inherit the residential house - No express clause within the will is needed re the residential house; the law of survivorship achieves testator’s aim - An express clause giving the share of the holiday home to the sister is required as this interest will otherwise pass as part of the gift of residue to the testator’s children)

How well did you know this?
1
Not at all
2
3
4
5
Perfectly
32
Q

A testator made a valid will 2 years ago that contains the following clause:

“I give to my nephew absolutely and free of tax and costs of transfer all my personal chattels as defined by section 55(1)(x) of the Administration of Estates Act 1925”

The residue of the estate passes to the testator’s brother. There are no other relevant clauses within the will.

The testator died yesterday. Included among their assets is a van which the testator used only for business purposes, cash in the house worth £35 and a ring that was purchased as an investment. The testator wore the ring on special occasions a few times each year.

Which of the following is correct?

A) The testator’s nephew will receive the ring. The testator’s brother will receive the £35 cash and the van.

B) The testator’s brother will receive the ring, £35 cash and the van.

C) The testator’s nephew will receive the ring and the van. The testator’s brother will receive the £35 cash.

D) The testator’s nephew will receive the ring, £35 cash and the van.

E) The testator’s nephew will receive the van. The testator’s brother will receive the ring and £35 cash.

A

A) The testator’s nephew will receive the ring. The testator’s brother will receive the £35 cash and the van

(Correct. The definition of chattels excludes business assets (the van), cash (£35) and assets owned solely as an investment. The ring was not owned solely as an investment so will be included in the gift of chattels, the other items are not chattels so will pass under the gift of residue)

How well did you know this?
1
Not at all
2
3
4
5
Perfectly
33
Q

A testator leaves the following legacy in their valid will:

“I give the piano I own at the date of my death to my neighbour”

The testator’s estate will be subject to inheritance tax.

Which one of the following is correct?

A) The gift of the piano is made subject to tax but free of costs of transfer.

B) The executor may choose whether the tax and transfer costs are borne by the beneficiary or treated as a general administration expense.

C) The gift of the piano is made free of tax and costs of transfer.

D) The gift of the piano is made subject to tax and costs of transfer.

E) The gift of the piano is made free of tax but subject to the costs of transfer.

A

E) The gift of the piano is made free of tax but subject to the costs of transfer.

(Correct. Where a will is silent, a legacy in a will is made free of tax but subject to the costs of transfer.
The other options were incorrect because the general rules were incorrectly stated and the executors do not have a general discretionary power to decide where the costs fall)

How well did you know this?
1
Not at all
2
3
4
5
Perfectly
34
Q

A valid will contains the following gifts:

“I give my car to my sister”

“I give my collection of stamps to my brother”

“I give £100 to my cleaner”

The testator owned a blue car when the will was made but had replaced this with a red car by the time they died.

After the testator made the will they sold over half of the stamps in the collection but later added new purchases to the collection.

There are no other relevant provisions.

Which one of the following is correct?

A) The testator’s brother will receive only the stamps that were in the collection at the date of the will.

B) The gift to the testator’s sister will fail and she will receive nothing.

C) The testator’s cleaner at the date of the testator’s death will receive £100.

D) The testator’s sister will receive the red car.

E) The testator’s brother will receive only the new stamps purchased after the will was executed.

A

B) The gift to the testator’s sister will fail and she will receive nothing

(Correct. The use of the word “my” in the specific gift overrides the general rule in s.24 Wills Act 1837 and the property is identified as at the date of the will. As the testator no longer owns the blue car then this gift adeems and the testator’s sister receives nothing.
The other options were incorrect because:
The identify of people is established with reference to the date of the will
Gifts of collections comprise the assets that fall within the collection as at the date of the testator’s death whether acquired before or after the will was executed)

How well did you know this?
1
Not at all
2
3
4
5
Perfectly
35
Q

By her will a testator left a property, which was charged with a mortgage during her lifetime, to her son. The residue of the testator’s estate is left to her daughter.

The will is silent on the liability to pay any outstanding mortgage. The testator did not have a mortgage protection policy.

Which of the following is correct?

A) The mortgage must be repaid by the executors using the sale proceeds of the property before they distribute the balance to the testator’s son.

B) The testator’s son will inherit the property subject to the mortgage.

C) The burden of repaying the mortgage will be shared equally between the property and the residue of the estate.

D) The mortgage will be repaid from the residue of the estate.

E) The burden of repaying the mortgage will be shared proportionately according to value between the property and the residue of the estate.

A

B) The testator’s son will inherit the property subject to the mortgage

(Correct. The general rule under s.35 Administration of Estates Act 1925, which applies here in the absence of any express wording to the contrary, states that the property charged shall bear the primary liability for repayment.
The other options were incorrect because:
The executors are not required to sell the property - they can transfer the property to the beneficiary subject to the charge
The residue of the estate is not liable for repayment of the mortgage)

How well did you know this?
1
Not at all
2
3
4
5
Perfectly
36
Q

A valid will contains the following gift:

“I give £400 to each of my children who are alive at the date of my death contingent on them reaching the age of 21”

When the testator dies they have three children A (aged 24), B (aged 19) and C (aged 15).

Which of the children has a contingent interest under the will?

A) B and C only

B) C only

C) A only

D) None of them

E) All of them

A

A) B and C only

(Correct. The gift is made contingent on the children reaching the age of 21. A has satisfied this contingency by the time the testator dies so their gift vests immediately. The other two children have not satisfied the contingency so their gifts are contingent. That one is an adult and the other a minor makes no difference.
The other options were incorrect as A has a vested interest, and B and C are under the age of 21 so have not satisfied the contingency)

How well did you know this?
1
Not at all
2
3
4
5
Perfectly
37
Q

You are reviewing a will that includes a class gift to all of the testator’s grandchildren contingent on them reaching the age of 18. The will does not state when the class of ‘grandchildren’ closes.

When the testator made the will they had five grandchildren. When the testator died they had seven grandchildren, all of whom are currently minors.

When will the class of ‘grandchildren’ close?

A) As the testator’s will does not specify when the class of ‘grandchildren’ closes the clause fails for uncertainty.

B) The class is already closed and only includes the grandchildren the testator had when they died.

C) When it is no longer possible for any further grandchildren to be born.

D) When the first of her grandchildren reaches 18.

E) The class is already closed and only includes the grandchildren the testator had when they made the will.

A

D) When the first of her grandchildren reaches 18

(Correct. The class closing rules mean the class of beneficiaries is identified when the first beneficiary has a vested interest. This has not yet occurred as all of the testator’s grandchildren are under the age of 18. Once the first of them reaches 18 the class will close.
The other options were incorrect because:
- The clause will not fail for uncertainty (the class closing rules create certainty)
- No beneficiary has a vested interest at the date of her death
- Without express wording to suggest otherwise, the date of the will is not the correct reference point
- It is possible to close the class without having to wait for it to be impossible for further members to be born)

How well did you know this?
1
Not at all
2
3
4
5
Perfectly
38
Q

A testator recently died leaving a valid will that contains the following gifts:

“I give £40,000 to my sister”

“I leave the residue of my estate equally between my children”

The testator’s sister died a year ago and is survived by her only child (the testator’s niece). One of the testator’s three children died 5 years ago, leaving two of her own children (the testator’s grandchildren).

There are no other relevant provisions in the will.

Which one of the following correctly describes how the testator’s estate will be distributed?

A) The testator’s niece will receive £40,000. The testator’s two surviving children will each inherit 1/2 of the residue.

B) The testator’s two surviving children will each inherit 1/3 of the residue. The two grandchildren will each inherit 1/6 of the residue.

C) The testator’s two surviving children and two grandchildren each inherit 1/4 of the residue.

D) The testator’s two surviving children will each inherit 1/2 of the residue.

E) The testator’s niece will receive £40,000. The testator’s two surviving children will each inherit 1/3 of the residue. The two grandchildren will each inherit 1/6 of the residue.

A

B) The testator’s two surviving children will each inherit 1/3 of the residue. The two grandchildren will each inherit 1/6 of the residue.

(Correct. The gift to the testator’s sister will lapse because she predeceased.
S.33 Wills Act 1837 will not create an implied substitution in favour of the testator’s niece because the original gift was not made to the testator’s issue. S.33 does apply to the gift to the children. Each child would be entitled to a 1/3 share, but one child has pre-deceased the testator. Their 1/3 share will be inherited equally by their two children, who receive 1/6 each.
The other options were incorrect because: - The testator’s niece does not receive anything - The testator’s grandchildren are entitled to share between them the 1/3 of the residue their parent would otherwise have inherited)

How well did you know this?
1
Not at all
2
3
4
5
Perfectly
39
Q

Which of the following is the standard applied when assessing the claim of a surviving spouse or civil partner under the IPFDA 1975?

A) Such financial provision as it would be reasonable in all the circumstances for a husband or wife or civil partner to receive for his/her maintenance.

B) Such financial provision as it would be reasonable in all the circumstances for any applicant to receive for his/her maintenance.

C) Such financial provision as it would be reasonable in all the circumstances for a husband or wife or civil partner to receive whether or not that provision is required for his or her maintenance.

A

C) Such financial provision as it would be reasonable in all the circumstances for a husband or wife or civil partner to receive whether or not that provision is required for his or her maintenance.

(Correct: The surviving spouse/civil partner standard does not require the court to consider whether the provision is required for the applicant’s maintenance)

How well did you know this?
1
Not at all
2
3
4
5
Perfectly
40
Q

Which of the following is the court not expressly required to consider under section 3(1) IPFDA 1975?

A) The applicant’s age and the duration of their relationship with the deceased.

B) Any physical or mental disability of any applicant or beneficiary.

C) The financial resources and financial needs of any beneficiaries of the deceased’s estate.

D) The size and nature of the net estate of the deceased.

E) The financial resources and financial needs of any applicants.

A

A) The applicant’s age and the duration of their relationship with the deceased

(Correct: Although the court is required to consider any matter that they consider relevant in the circumstances, the applicant’s age and duration of their relationship with the deceased are not expressly referred to in section 3(1) IPFDA)

How well did you know this?
1
Not at all
2
3
4
5
Perfectly
41
Q

Which of the following most accurately reflects the guidelines applicable to spouses and civil partners?

A) The court must consider the applicant’s age and duration of the marriage or civil partnership, as well as any contributions made by the applicant to the welfare of the deceased (including any contributions made by looking after the home or caring for the family) and the liability of any other person to maintain the applicant. The court must also take into account what the applicant might reasonably have expected to receive on divorce or dissolution of the civil partnership.

B) The court must consider the applicant’s age and duration of the marriage or civil partnership, as well as any contributions made by the applicant to the welfare of the deceased (including any contributions made by looking after the home or caring for the family). The court must also take into account what the applicant might reasonably have expected to receive on divorce or dissolution of the civil partnership.

C) The court must consider the applicant’s age and duration of the marriage or civil partnership, as well as any contributions made by the applicant to the welfare of the deceased (including any contributions made by looking after the home or caring for the family) and whether, and if so to what extent, the deceased assumed responsibility for the maintenance of the applicant. The court must also take into account what the applicant might reasonably have expected to receive on divorce or dissolution of the civil partnership.

D) The court must consider the applicant’s age and duration of the marriage or civil partnership, as well as whether, and if so the length of time for which and the basis on which the deceased maintained the applicant, and the extent of the contribution made by way of maintenance. The court must also take into account what the applicant might reasonably have expected to receive on divorce or dissolution of the civil partnership.

E) The court must consider the applicant’s age and duration of the marriage or civil partnership, as well as the length of any period of cohabitation prior to the marriage or civil partnership. The court also considers the contributions made by the applicant to the welfare of the deceased (including any contributions made by looking after the home or caring for the family). The court must also take into account what the applicant might reasonably have expected to receive on divorce or dissolution of the civil partnership.

A

B) The court must consider the applicant’s age and duration of the marriage or civil partnership, as well as any contributions made by the applicant to the welfare of the deceased (including any contributions made by looking after the home or caring for the family). The court must also take into account what the applicant might reasonably have expected to receive on divorce or dissolution of the civil partnership

(Correct: The court considers all the factors above. There is no express requirement to consider whether the deceased maintained the applicant, or the length of their relationship prior to marriage/civil partnership (although these may be considered under the common guidelines if the court considers them relevant in the circumstances).)

How well did you know this?
1
Not at all
2
3
4
5
Perfectly
42
Q

A client seeks advice with regards the following clause in her father’s will.

“I give all of my chattels to my daughter and hope that she will distribute them in accordance with the instructions I have left with my will”.

There is a letter of instruction attached to the father’s will requesting that your client distributes his chattels equally between all of his grandchildren as she sees fit.

Which one of the following is correct?

A) If the client distributes the chattels to her children she would make a PET for IHT purposes.

B) By this clause the client is appointed as trustee of the chattels.

C) The client has an obligation to distribute the chattels to her children.

D) The effect of s 143 IHTA is to treat the gift of chattels to the client as being made free of IHT.

E) If the client distributes the chattels to her children she would make a disposal for CGT purposes.

A

E) If the client distributes the chattels to her children she would make a disposal for CGT purposes.

(Correct. The clause in the will is a precatory trust. The client is the beneficiary of the chattels. However, her father hoped that she would distribute them to his grandchildren. No legal trust is created so the client is not a trustee and has no obligation to comply with her father’s wishes. If she does give the chattels to the grandchildren, s 143 IHTA applies automatically to treat these gifts as if they were made by her father’s will (and not the client) for IHT purposes. There is no equivalent for CGT however, so the gifts will be a disposal by the client for CGT purposes.
The other options were incorrect because:
S 143 does not create a tax-free gift under the will
A gift of chattels made by the client to the grandchildren would ordinarily be a PET but the effect of s 143 is to treat these gifts as having been made by the will of the client’s father, and so she will not make a PET
No formal trust is created
The client has no obligation to comply with the wishes of her fathe)

How well did you know this?
1
Not at all
2
3
4
5
Perfectly
43
Q

A client seeks your advice following the death of her brother 18 months ago. By her brother’s will your client is due to inherit the whole of his estate, with a substitution in favour of her children if she did not inherit for any reason.

The client is unhappy that her brother did not include their step-sister in his will and now wants to change the distribution of his estate so that she keeps half and the other half passes to their step-sister.

The client does not want to be taxed for either IHT or CGT purposes as though she personally made this gift to her step-sister.

Which of the following is correct?

A) The client’s objectives can be met if she enters either a variation or disclaimer because the writing back effect for IHT and CGT purposes applies to both.

B) The client’s objectives can only be met if she disclaims her inheritance.

C) The client’s objectives can only be met if she varies her inheritance.

D) It is not possible for all of the client’s objectives to be met by either a variation or disclaimer.

E) It is not possible for the client to enter into either a variation or disclaimer because her brother died more than a year ago.

A

C) The client’s objectives can only be met if she varies her inheritance.

(Correct. A variation would achieve all of her objectives. By way of variation the client can re-distribute part of her inheritance and elect who should benefit instead. The writing back effect for both IHT and CGT will apply provided the variation is made in writing within 2 years following her brother’s death. The other options were incorrect because:
A disclaimer could not be used because the client cannot disclaim part of her inheritance, neither can she nominate the alternative beneficiary (under the will any disclaimed inheritance would pass directly to her children, not the step sister)
The time limit for varying the distribution is 2 years following death)

How well did you know this?
1
Not at all
2
3
4
5
Perfectly
44
Q

A client seeks your advice following the death of their father. The client recently inherited a share of her father’s estate but does not wish to keep her inheritance because she is wealthy in her own right. Instead your client wishes to pass part of her inheritance (a cash sum of £50,000) to her adult children in the most tax efficient manner. Inheritance tax was payable following her father’s death. You advise your client to enter a deed of variation.

Which of the following is the best advice for your client?

A) Your client should make an election for s 142 IHTA to apply to the variation.

B) Your client should make an election for s 142 IHTA and s 62 TCGA to apply to the variation.

C) There is no benefit to claiming the writing back effect under either s 142 IHTA or s 69 TCGA because no refund of tax can be claimed.

D) Your client should make an election for s 62 TCGA to apply to the variation.

E) Your client will need to seek the approval of her father’s PRs before she can vary her inheritance.

A

A) Your client should make an election for s 142 IHTA to apply to the variation.

(Correct. If the client enters a variation and claims the writing back effect for IHT under s 142 IHTA her father’s estate will be taxed as though £50,000 was given to the client’s children and the remainder to the client. There is no change to the IHT liability on her father’s estate as a result (none of the beneficiaries are exempt) therefore consent from her father’s PRs is not required. The advantage to claiming the writing back effect under s 142 is that the client will not be making PET and so avoids the risk of this failing if she died within 7 years.
The other options were incorrect because:
No writing back for capital gains tax purposes should be made because the subject matter of the variation is cash- which is exempt from CGT.
There is an advantage to making an election under s 142 IHTA – the gift of £50,000 to the client’s children is treated as having been made from her father’s estate and will not be a PET by the client.
There would be no change to the IHT liability following the variation so PR consent is not required.)

How well did you know this?
1
Not at all
2
3
4
5
Perfectly
45
Q

A man died and by his valid will left the whole of his estate to his adult children.

The man’s civil partner died before the man and left the man the whole of his estate.

Neither the man nor his civil partner had made any lifetime transfers.

The man’s estate was valued at £1.5 million, which includes his home worth £500,000.

What is the value of the inheritance tax nil rate band available for the man’s estate?

A) £500,000

B) £1,000,000

C) £325,000

D) £825,000

E) £650,000

A

B) £1,000,000

(Correct.
The man has his own NRB of £325,000.
The man’s PRs can also claim the TNRB in respect of his civil partner who died before him. We know that the man’s civil partner left all of his estate to the man. Therefore, the man’s civil partner did not use his own NRB because his whole estate qualified for civil partner exemption.
The RNRB of £175,000 also applies. The man left a qualifying residential interest (his home) to his lineal descendants (his children). As the man’s civil partner did not use his own RNRB, this can also be claimed by the man.
As neither of them made any lifetime transfers the maximum amounts are available.)

How well did you know this?
1
Not at all
2
3
4
5
Perfectly
46
Q

A man died yesterday leaving his whole estate to a life interest trust. The man’s spouse and children survive him. The man’s spouse is the life tenant of his will trust and his children are the remaindermen.

The man had not made any lifetime gifts.

His estate comprised his home worth £500,000 (free of mortgage) and various bank accounts totalling £120,000. The man’s debts and funeral expenses totalled £8,000.

In the tax year of the man’s death, the basic nil rate band is £325,000 and the main residence nil-rate band is £175,000.

How much inheritance tax will be payable on the man’s estate?

A) £78,000

B) £244,800

C) £114,800

D) £0

E) £130,000

A

D) £0

(Correct: The whole of the man’s estate passes to a life interest trust in which his spouse is the life tenant. Spouse exemption applies to the whole estate and no inheritance tax is payable. The other options were incorrect because spouse exemption was not applied)

How well did you know this?
1
Not at all
2
3
4
5
Perfectly
47
Q

A woman died and by her valid will left the whole of her estate to her adult children. The woman’s civil partner died before the woman and left the woman the whole of her estate.

Neither the woman nor her civil partner had made any lifetime transfers.

The woman’s home is worth £700,000 and she had savings of £400,000 and other assets worth £1.6 million.

What is the value of the inheritance tax nil rate band available for the woman’s estate?

A) £325,000

B) £650,000

C) £825,000

D) £1,000,000

E) £500,000

A

B) £650,000

(Correct.
The woman has her own NRB of £325,000.
The woman’s PRs can also claim the TNRB in respect of her civil partner who died before her. We know that the woman’s civil partner left all of her estate to the woman. Therefore, the woman’s civil partner did not use her own NRB because her whole estate qualified for civil partner exemption.
As neither made any lifetime transfers the maximum amounts are available.
The RNRB does not apply as the total value of the woman’s estate is above the threshold)

How well did you know this?
1
Not at all
2
3
4
5
Perfectly
48
Q

A man died yesterday leaving his whole estate to his son. The man had never married or entered a civil partnership and had not made any lifetime gifts.

His estate comprised his home worth £150,000 (free of mortgage) which he had lived in for over 20 years and various bank accounts totalling £340,000. He also owned a house worth £180,000 free of mortgage which was rented out to tenants and had never been his residence.

The man’s debts and funeral expenses totalled £20,000.

In the tax year of the man’s death, the basic nil rate band is £325,000 and the main residence nil-rate band is £175,000.

How much inheritance tax will be payable on the man’s estate?

A) £70,000

B) £130,000

C) £0

D) £78,000

E) £60,000

A

A) £70,000

(Step 1: The man’s cumulative total was 0 as he made no lifetime gifts.
Steps 2/3: The man’s taxable estate includes both properties and the bank accounts with a total of £670,000.
Step 4: The man’s debts and funeral costs which total £20,000 can be deducted, leaving £650,000.
Step 5: No exemptions or reliefs apply.
Step 6: The man can claim his own RNRB. No transferrable RNRB applies as he does not have a spouse who pre-deceased. His main residence is a QRI and it passes to his son (a lineal descendent) absolutely. The amount of the RNRB is £150,000 (capped at the value of the QRI). The let property is not a QRI so an RNRB can be claimed. The total RNRB is: £150,000. Once applied to his estate, £650,000 - £150,000 = £500,000 remains.
Step 7: The man’s basic NRB is £325,000 (his cumulative total was 0 – so no deduction is made). No transferrable NRB applies as he did not have a pre-deceasing spouse.
0 - £325,000 @ 0%. Balance of £175,000 (£500,000 - £325,000) @ 40% = £70,000)

How well did you know this?
1
Not at all
2
3
4
5
Perfectly
49
Q

A property was purchased as tenants in common by an unmarried couple in unequal shores. The woman owned 80% and the man owned 20%. The woman died last month and by her will leaves her share of the property to her son absolutely.

The current market value of the property is £100,000. There is no mortgage.

The woman’s estate will be subject to inheritance tax (IHT).

What will be the value of woman’s share of the property for IHT purposes?

A) £0 as the property passes by survivorship.

B) £80,000 - no discount is available because the couple were not married.

C) £50,000 less a discount to reflect the fact that that the property was co-owned.

D) £80,000 less a discount to reflect the fact that the property was co-owned.

E) £80,000 - no discount is available because the related property rules apply.

A

D) £80,000 less a discount to reflect the fact that the property was co-owned.

(Correct: The value of the woman’s share is £80,000 (she owned 80% of the total worth £100,000). The woman co-owned the property and a discount to reflect the fact that the property was co-owned can be applied.
The other options were incorrect because:
- The property does not pass by survivorship, but in any event, the value of the woman’s share is taxable whether owned as joint tenants or tenants in common
- The value of her share is proportionate to share she owned, not the number of co-owners. A 50/50 split applies to joint tenants and those owning as tenants in common in equal shares.
- The related property rules apply to married couples and are not relevant here.
- A discount is available because the couple were not married. It would not have been available if they were)

How well did you know this?
1
Not at all
2
3
4
5
Perfectly
50
Q

Is the following statement True or False?

“Where an executor is appointed by the will their power to act as PR derives from this appointment. Therefore, executors will not usually need to apply for a grant of representation.”

A

False

The authority for an executor to act as PR does derive from the will, whereas the authority for an administrator to act derives from the grant of representation. However, an executor will still need to apply for a grant of representation because many institutions will insist on seeing the grant as proof of the executor’s authority to act before releasing funds/assets to them.

How well did you know this?
1
Not at all
2
3
4
5
Perfectly
51
Q

Is the following statement True or False?

“Estate Administration is the process of collecting in the deceased’s assets and distributing them to the correct beneficiaries”.

A

False
Correct
This was an incomplete statement. The administration process also includes other obligations, including payment of the deceased’s debts and administration expenses.

How well did you know this?
1
Not at all
2
3
4
5
Perfectly
52
Q

Is the following statement True or False?

“Once the grant of representation is obtained the PRs have authority to collect in and deal with all the assets that the deceased owned when they died.”

A

False
Correct
The grant of representation confers power on the PRs to deal with the succession estate assets, ie those items passing under the will or under intestacy, and does not, for example, give PRs the right to have assets passing by survivorship transferred into their name.

How well did you know this?
1
Not at all
2
3
4
5
Perfectly
53
Q

True/False: An executor may also be a trustee of the estate they administer.

False

True

A

True

The executor of an estate may be expressly appointed in the role of trustee of any trust that arises following death, but that this is not always the case.

How well did you know this?
1
Not at all
2
3
4
5
Perfectly
54
Q

True/False: Acting as a PR confers a fiduciary duty and therefore a beneficiary should not act as PR because this will give rise to a conflict of interest.

False

True

A

False

The role of a PR is fiduciary in nature but there is no prohibition on a PR also being a beneficiary. There is no conflict of interest as the PR does not receive benefit in their capacity as PR.

How well did you know this?
1
Not at all
2
3
4
5
Perfectly
55
Q

True/False: A solicitor may be involved with the estate administration even if they have not been appointed as PR.

True

False

A

True

(A solicitor may become involved if they are appointed as PR by a will or if the PRs instruct a solicitor to carry out administrative services)

How well did you know this?
1
Not at all
2
3
4
5
Perfectly
56
Q

Is the following statement true or false?

“If an estate has not been fully administered 12 months after the deceased’s death the PRs will be in breach of their duty under s.44 Administration of Estates Act 1925?”

A

False

(Correct. There is no duty to complete the administration within the “executor’s year”. A PR is not obliged to distribute the estate before the one-year anniversary of death, although after this period they must be able to justify any delay)

How well did you know this?
1
Not at all
2
3
4
5
Perfectly
57
Q

Your client will be appointed as the PR of an estate. Your client has not yet made an application for a grant.

Which of the following most accurately describes the current statutory duties of your client?

A) A duty to report to HMRC about the estate assets and liabilities and pay any inheritance tax due and administer the estate according to the law.

B) There are no statutory duties that apply until the grant has been issued.

C) A duty to collect in the deceased’s assets, administer them according to law and keep estate accounts.

D) A duty to collect in the deceased’s assets and administer them according to law.

E) A duty to report to HMRC about the estate assets and liabilities and pay any inheritance tax due.

A

E) A duty to report to HMRC about the estate assets and liabilities and pay any inheritance tax due.

(Correct. Prior to the grant the PR is not able to collect in and administer the deceased’s assets and the primary duty the PR has is to report to HMRC and pay the IHT, both of which must be completed before the grant is issued)

How well did you know this?
1
Not at all
2
3
4
5
Perfectly
58
Q

You are advising a client who has been appointed the sole executor of his fathers estate. The executor has just obtained the grant of probate and does not act in a professional capacity.

The executor has asked whether he must complete the administration of the estate now he has started and whether he may buy one of the properties from the estate.

There are no relevant express provisions in the will.

Which of the following is correct?

A) The executor has a duty to administer the estate. The executor has the power as PR to sell estate assets. If he acted as the purchaser he would be in breach of his fiduciary duty, even if a fair price was paid (unless consent of the court or beneficiaries was obtained).

B) The executor has a duty to administer the estate. The executor has the power as PR to sell estate assets and may act as the purchaser provided the price paid was full market value and there is no loss to the estate.

C) The executor does not have to administer the estate but any administrative steps he does take must be done with due diligence. The executor has the power as PR to sell estate assets but if he acted as the purchaser, he would be in breach of his fiduciary duty even if a fair price was paid.

D) The executor does not have to administer the estate but any administrative steps he does take must be done with due diligence. The executor has the power as PR to sell estate assets and may act as the purchaser provided the price paid was full market value.

E) The executor has a duty to administer the estate. The executor has the power as PR to sell estate assets and fiduciary duties do not apply because he is not acting as a trustee.

A

A) The executor has a duty to administer the estate. The executor has the power as PR to sell estate assets. If he acted as the purchaser he would be in breach of his fiduciary duty, even if a fair price was paid (unless consent of the court or beneficiaries was obtained)

(Correct. An executor who accepts office has a duty to administer the estate in full under s.25 AEA. A PR is subject to fiduciary duties which include the duty to avoid a position of conflict. Acting as both buyer and seller of the estate assets (without court or beneficiary authorisation) would place him in a position of conflict and would therefore breach that duty, even if there was no loss suffered to the estate and fair market value was paid)

How well did you know this?
1
Not at all
2
3
4
5
Perfectly
59
Q

A testator died yesterday leaving an estate worth £3 million. The assets include £1 million cash in a current account (being the sale proceeds from a recent property sale). The current rate of interest is very low.

The PRs are concerned about their obligations in relation to the estate assets and believe the administration will not be completed for some time.

Which of the following statements is correct?

A) The PRs should retain the bank account as it is because this was the original asset owned by the deceased.

B) The PRs would only be subject to the statutory duty of care when exercising their power to invest the estate assets if they were acting in capacity as a trustee.

C) The PRs have a duty to invest the cash in the bank account to achieve a greater rate of return than the current rate of interest.

D) The statutory power to invest does not apply to the PRs unless an express trust has been created by the testator’s will.

E) The PRs have complete discretion to invest cash in the bank account in any kind of assets as if they were absolutely entitled.

A

C) The PRs have a duty to invest the cash in the bank account to achieve a greater rate of return than the current rate of interest

(Correct. As the administration is likely to take time the PRs have a duty to maintain the estate and actively invest to achieve better rates of return than are being paid in respect of the current account.
The other answers were incorrect because: - Although the general power of investment gives the PRs the power to invest as if they were absolutely entitled, their discretion is not absolute. They must invest having regard to the standard investment criteria and must seek and follow appropriate advice. - The statutory power to invest (s.3 Trustee Act 2000) does apply to PRs - There is no obligation to retain original assets - The statutory duty of care applies to the exercise of investment powers and is applicable to PRs as well as trustees)

How well did you know this?
1
Not at all
2
3
4
5
Perfectly
60
Q

A testator died recently leaving a cash legacy of £10,000 to his friend.

The executors have identified a small portfolio of quoted shares in the estate currently worth £8,000 that they would like to transfer to the friend in part satisfaction of his legacy. The shares have not been specifically gifted to anyone under the will and the residue of the estate passes to the testator’s spouse.

The will does not contain any express administrative provisions.

Assuming the shares do not subsequently increase in value to more than £10,000, which of the following would be the most appropriate advice for the executors?

A) Transfer the shares to the friend.

B) Transfer the shares to the friend plus a cash sum of £2,000.

C) Transfer the shares to the friend, but only if the spouse consents.

D) Transfer £10,000 to the friend. They have no power to transfer shares in partial satisfaction of the cash legacy.

E) Transfer the shares to the friend plus a cash sum equivalent to the difference in value between the shares (at the date of appropriation) and £10,000.

A

E) Transfer the shares to the friend plus a cash sum equivalent to the difference in value between the shares (at the date of appropriation) and £10,000.

(Correct. S.41 AEA provides the PRs with the power to appropriate assets in or towards a beneficiary’s entitlement. As the value of the shares is less than the legacy the PRs may appropriate the shares and then make a balancing cash transfer of the difference. The consent of the residuary beneficiary is not required as the shares were not specifically given to them)

How well did you know this?
1
Not at all
2
3
4
5
Perfectly
61
Q

By her will a testator left a legacy of £500 to her granddaughter (G), who was 16 at the date of T’s death. The will does not amend any of the statutory powers or include any express administrative provisions.

Assuming T’s PRs wish to distribute this amount as soon as possible, which of the following is the most appropriate course of action?

A) The PRs must wait until G reaches the age of 18 or marries before distributing the legacy.

B) The PRs must wait until G reaches the age of 18 before distributing the legacy.

C) The PRs cannot give the money to G directly but can appoint trustees of the legacy and pay the money to the trustees.

D) The PRs have discretion to pay the money to G if they consider she is responsible enough to receive the money direct.

E) The PRs have the power to pay the money to G because she is over the age of 16.

A

C) The PRs cannot give the money to G directly but can appoint trustees of the legacy and pay the money to the trustees

(Correct. By s.42 Administration of Estes Act 1925 the PRs may appoint trustees of a legacy and pay the amount of the legacy to the trustees, who hold on trust for the minor beneficiary. The trustees will provide confirmation of the receipt to the PRs and so the administration can be completed.
The other options were incorrect because: - There is no discretion to pay the money directly to a minor - The PRs do not have to wait until the minor reaches 18 or married - The power to pay the money to a beneficiary aged 16 or older must be conferred by the will and in this scenario we know that the will does not contain any administrative provisions)

How well did you know this?
1
Not at all
2
3
4
5
Perfectly
62
Q

You are advising a creditor of an estate who is concerned they will not receive money they are owed.

Assuming there are sufficient funds available to meet all debts and legacies but the creditor is not paid, which of the following is correct?

A) There is no cause of action that can be brought against the PRs as their duties are only owed to the beneficiaries of an estate.

B) The creditor may only bring a claim against the PRs if the administration of the estate is not yet complete and enough money remains to pay the debt.

C) If the creditor cannot bring a claim against the PRs there is no alternative option for them to pursue.

D) The creditor may bring a devastavit claim against the PRs on the grounds of maladministration.

E) The creditor may bring a claim against the PRs on the grounds of breach of fiduciary duty.

A

D) The creditor may bring a devastavit claim against the PRs on the grounds of maladministration

(Correct. A claim against a PR is referred to as a devastavit. Where the estate has not been distributed to those who are entitled to it the grounds for a claim would be maladministration.
The other options were incorrect because:
- Duties are owed to both beneficiaries and creditors
- The wrongdoing would not amount to a breach of fiduciary duty
- An unpaid creditor could also claim against the estate beneficiaries
- The PRs owe a personal liability to a creditor who was not paid from the estate)

How well did you know this?
1
Not at all
2
3
4
5
Perfectly
63
Q

You are advising a lay executor about their duties. The deceased left a will that includes an express clause stating “no executor shall be personally liable for any action or inaction taken in the course of the administration”.

Which of the following is correct?

A) The clause is effective and the executor will not have any personal liability.

B) The clause is not effective as it is not possible to limit the liability of an executor.

C) The clause is not required because the executor is not acting in a professional capacity.

D) The clause would not protect the executor from loss resulting from an honest mistake.

E) Despite the express clause in the will the executor would remain liable for fraud.

A

E) Despite the express clause in the will the executor would remain liable for fraud.

(Correct. A clause which limits the executor’s liability is enforceable but not to the extent it attempts to avoid liability for fraud.
The other options were incorrect because:
- it is not possible to remove all personal liability irrespective of how the executor behaves.
- both lay and professional executors have personal liability
- limitation of liability clauses are enforceable in respect of an honest mistak)

How well did you know this?
1
Not at all
2
3
4
5
Perfectly
64
Q

The PRs of an estate are due to distribute assets to the deceased’s children. They have been in contact with the children of the deceased’s marriage but are concerned that the testator could have fathered other children who the PRs and immediate family are unaware of.

Which of the following would help protect the PRs from personal liability should any additional children approach the PRs for a share of the estate after it has been distributed?

A) Compliance with the s.27 Trustee Act 1925 notice procedure.

B) Application to court for an administration action.

C) Retaining funds and delaying the completion of the administration indefinitely.

D) Making a payment into court.

E) Application to court for a Benjamin Order.

A

A) Compliance with the s.27 Trustee Act 1925 notice procedure

(Correct. S.27 notices are intended to help protect the PRs from personal liability to unknown creditors and beneficiaries. The other options are possible steps a PR might take if they know about a beneficiary but cannot locate them)

How well did you know this?
1
Not at all
2
3
4
5
Perfectly
65
Q

A deceased died leaving a valid will that appoints their brother and sister as joint executors.

The will gives all of the estate to the deceased’s brother.

The deceased is survived by their sister and she is willing to act. The deceased’s brother pre-deceased him so some of the estate will pass under the intestacy rules.

Which is the correct grant?

A) Grant of Representation

B) Grant of double probate

C) Grant of letters of administration (with will)

D) Grant of letters of administration

E) Grant of probate

A

E) Grant of probate

(Correct. The deceased left a will that appoints an executor. It does not matter that one of the executors has pre-deceased, the remaining executor can apply. It does not matter that the will fails to dispose of all of the deceased’s assets)

How well did you know this?
1
Not at all
2
3
4
5
Perfectly
66
Q

A testator died leaving a valid will appointing their father as executor. The will gives all of the estate to the testator’s brother. The testator is survived by their brother, but their father has pre-deceased.

Which is the correct grant?

A) Grant of probate

B) Grant of letters of administration (with will)

C) Grant of Representation

D) Grant of letters of administration

E) Grant of double probate

A

B) Grant of letters of administration (with will)

(Correct. The deceased left a will which does not appoint any executors who can act)

How well did you know this?
1
Not at all
2
3
4
5
Perfectly
67
Q

A deceased died leaving a valid will that appoints their brother as executor. The deceased’s will leaves a property to their spouse and the rest of the estate to their two minor children.

The deceased is survived by their brother and children. Their spouse has pre-deceased and the deceased has died partially intestate.

Which is the correct grant?

A) Grant of Representation

B) Grant of letters of administration

C) Grant of letters of administration (with will)

D) Grant of probate

E) Grant of double probate

A

D) Grant of probate

(Correct. The deceased left a will that appoints an executor. It does not matter that the deceased died partially intestate and only one executor is required even where minor interests arises)

How well did you know this?
1
Not at all
2
3
4
5
Perfectly
68
Q

You are instructed to assist with the administration of an estate which contains the following assets:

- Property owned as tenants in common

- Bank account with £60,000

- Furniture

- Life policy where proceeds have been nominated for the deceased’s spouse

Which of the following correctly states whether or not a grant of representation would be required to deal with the assets?

A) A grant is required to deal with the property and the bank account only.

B) A grant is required to deal with the property and the life policy only.

C) A grant is required to deal with the bank account, furniture, and life policy proceeds only.

D) The estate could be administered without the need for a grant.

E) A grant is required to deal with all of the assets.

A

A) A grant is required to deal with the property and the bank account only

(Correct. The life policy proceeds do not fall within the succession estate and may be paid directly to the deceased’s spouse and no grant is required to deal with household possessions. The deceased’s share of property owned as tenants in common will pass as part of the succession estate and a grant is required. As there is more than small amount in the bank account this would usually require a grant to be provided before the bank would release the funds)

69
Q

You are instructed to assist with the administration of an estate which contains the following assets:

**- Property owned as tenants in common(*

- Building Society account with £2,000

- Furniture

- Life policy where proceeds of £100,000 are payable to the estate

Which of the following correctly states whether or not a grant of representation is needed to administer these assets?

A) A grant is required to deal with all of the assets.

B) The estate could be administered without the need for a grant.

C) A grant is required to deal with the property and the life policy only.

D) A grant is required to deal with the building society account, furniture, and life policy proceeds only.

E) A grant is required to deal with the property, building society account and life policy proceeds only.

A

C) A grant is required to deal with the property and the life policy only

(Correct. A grant is required to deal with assets within the succession estate unless an exception applies. A property owned as tenants in common, and a life policy where proceeds are payable to the estate, are items that fall within the succession estate and will require the PRs to take out a grant to deal with them. The account with a balance of less than £5,000 could be collected under the exception in the Administration of Estates (Small Payments) Act 1965 and no grant is required to deal with household possessions)

70
Q

You are instructed to assist with the administration of an estate which contains the following assets:

- Property owned as joint tenants - Building society account with £2,000 - Furniture - Life policy where proceeds have been nominated for the deceased’s spouse

Which of the following correctly states whether or not a grant of representation would be required to deal with the assets?

A) A grant is required to deal with all of the assets.

B) A grant is required to deal with the property and the life policy only.

C) The estate could be administered without the need for a grant.

D) A grant is required to deal with the property, building society account and life policy proceeds only.

E) A grant is required to deal with the building society account, furniture and life policy proceeds only.

A

C) The estate could be administered without the need for a grant

(Correct. A property owned as joint tenants does not fall within the succession estate and passes under survivorship rules. The life policy proceeds do not fall within the succession estate and may be paid directly to the deceased’s spouse. The building society account with a balance of less than £5,000 could be collected under the Administration of Estates (Small Payments) Act 1965 and no grant is required to deal with household possessions)

71
Q

A deceased owned the following assets among their personal possessions:

- Dining table (worth approximately £200 if sold but would cost £1,500 to replace)

- Painting worth approximately £300 (sale and replacement value are similar)

- Diamond ring worth approximately £6,000 (sale and replacement value are similar)

Regarding the value of the assets, which of the following would be the most appropriate?

A) The PRs should obtain a professional valuation for the ring but may record estimated values for the dining table and the painting.

B) The PRs should obtain a professional valuation for the dining table and the painting but may record an estimated value for the ring.

C) The PRs should obtain a professional valuation for the dining table and ring but may record an estimated value for the painting.

D) The PRs should obtain a professional valuation for all of the assets.

E) The PRs may record estimated values for all of the assets.

A

A) The PRs should obtain a professional valuation for the ring but may record estimated values for the dining table and the painting.

(Correct. The sale value of the assets at the date of death is relevant. Estimated figures may be used for ‘low value’ chattels, which would include the dining table (£200 sale value) and the painting (£300). The ring is a single item worth more than £500 so a professional valuation should be obtained)

72
Q

You have been appointed as an executor of an estate jointly with the deceased’s adult son. The deceased’s assets comprised a car, a bank account and a small partnership interest in their sole name. The deceased owned his home and personal chattels as joint tenants with his spouse.

The deceased’s surviving spouse has already registered the death and is currently arranging the funeral.

Which of one of the following does not need to be carried out by the executors?

A) Transferring the house and chattels into the name of the executors.

B) Arranging a professional valuation of the business.

C) Locating the original will.

D) Notifying assets holders of the deceased’s death.

E) Valuing the deceased’s debts.

A

A) Transferring the house and chattels into the name of the executors

(Correct. The joint tenant property passes automatically to the survivor under the survivorship rules and does not form part of the succession estate for administration purposes and would not be transferred into the names of the executors)

73
Q

You have been instructed by the executors of an estate to assist with the administration.

The deceased owned a property jointly with his brother and held a joint bank account with his son.

When preparing a schedule of assets in respect of the deceased’s estate which of the following is correct?

A) The deceased will be deemed to own 50% of the bank account. Further enquiries are needed to establish whether or not the deceased held a 50% share of the property.

B) Further enquiries are needed to establish whether or not the deceased held a 50% share of the property and joint bank account.

C) The deceased will be deemed to own 50% of the property and 100% of the bank account (because this passes by survivorship).

D) The deceased will be deemed to own 50% of both the property and the joint bank account.

E) The deceased will be deemed to own 50% of the property. Further enquiries are needed to establish whether or not the deceased held a 50% share of the bank account.

A

B) Further enquiries are needed to establish whether or not the deceased held a 50% share of the property and joint bank account.

(Correct. Where an asset is owned jointly (whether a property or bank account) enquiries are needed to establish what the terms of that co-ownership are. You cannot assume that the deceased held a 50% share or that all joint property is owned as joint tenants.
The other options were incorrect because it was assumed that the deceased owned 50% of one or both of the assets)

74
Q

For which of the following estates might the chain of representation apply?

• Testator A appoints a sole executor who survives A but dies before taking out the grant in A’s estate.

• Testator B appoints two executors, one of whom dies after taking out the grant in B’s estate. The other wishes to act.

• Testator C appoints an executor who has died before C.

• Testator D appoints an executor who dies intestate after taking out the grant in D’s estate.

A) A’s estate

B) B’s estate

C) None of them

D) C’s estate

E) D’s estate

A

C) None of them

(Correct. The chain of representation only applies where a sole surviving executor, who has taken out a grant of probate, dies before administering the estate and appoints their own executor in turn.
The others were incorrect because:
Estate A: the executor died before taking the grant. Estate B: there is a surviving executor who will act Estate C: the executor did not survive the testator Estate D: the sole surviving executor died intestate so cannot have appointed an executor for their own estate)

75
Q

A testator died leaving a will that appoints their civil partner, sister and brother jointly as executors.

The testator’s civil partnership was subsequently dissolved and the testator’s sister has pre-deceased.

Which of the following correctly states who is entitled to apply for the grant of representation?

A) The brother or civil partner

B) The brother

C) The brother and the sister’s own executor

D) The brother or the sister’s own executor

E) The brother and civil partner

A

B) The brother

(Correct. The civil partner cannot act because of the effect of s.18C Wills Act 1837. The sister’s executor cannot act because the chain of representation does not apply in this scenario. The brother may apply as the sole surviving executor)

76
Q

A testator died leaving a will that appoints A, B, C, D and E as joint executors.

A is aged 17. B, C, D and E are adults.

Which of the following most accurately explains who will take out the grant of probate?

A) B, C, D and E. Power may be reserved to A who could apply once they reach the age of 18.

B) A, B, C, D as the first named executors to the maximum of four.

C) B, C , D and E. Power cannot be reserved to A because they are a minor.

D) B, C, D and E. A cannot act because an appointment of a minor is invalid.

E) A, B, C, D and E

A

A) B, C, D and E. Power may be reserved to A who could apply once they reach the age of 18

(Correct. A minor may be appointed by will but cannot act until they reach the age of 18. Power may be reserved to them until this time. A maximum of four executors may apply, so B, C, D and E can be named on the grant)

77
Q

A testator died leaving a will that appoints A, B, C, D and E as joint executors.

A is aged 17. B, C, D and E are adults.

Which of the following most accurately explains who will take out the grant of probate?

A) B, C, D and E. Power may be reserved to A who could apply once they reach the age of 18.

B) A, B, C, D as the first named executors to the maximum of four.

C) B, C , D and E. Power cannot be reserved to A because they are a minor.

D) B, C, D and E. A cannot act because an appointment of a minor is invalid.

E) A, B, C, D and E

A

A) B, C, D and E. Power may be reserved to A who could apply once they reach the age of 18

(Correct. A minor may be appointed by will but cannot act until they reach the age of 18. Power may be reserved to them until this time. A maximum of four executors may apply, so B, C, D and E can be named on the grant)

78
Q

A testator made a valid will appointing their civil partner, mother and son (17) jointly as executors.

The testator’s civil partnership was dissolved after the will was executed.

The testator’s mother does not have mental capacity.

There are no relevant express provisions in the will.

Who should apply for a grant of probate?

A) The mother and the son

B) The civil partner only

C) None of the executors named in the will

D) All three executors named in the will

E) Civil partner and the mother only

A

C) None of the executors named in the will

(Correct. The testator’s former civil partner cannot act (s.18C Wills Act 1837) unless the will expressly states otherwise (which it does not). A minor and someone who lacks capacity are both unable to act)

79
Q

A deceased died intestate and their estate passes to their spouse and two children (one of whom is a minor).

The spouse has not applied for a grant but has been dealing with the estate administration and has arranged for the sale of the deceased’s property. However, the spouse no longer wishes to be involved with the administration.

Which of the following would you advise?

A) Doing nothing

B) Acting alone

C) Renunciation

D) Reserving power

A

C) Renunciation

(Correct. The spouse with an entitlement to apply as administrator may renounce that right even if she has intermeddled. The other options were incorrect because: - Reserving power is not an option for administrators - She cannot act alone where there is a minor interest - Doing nothing is not a viable option)

80
Q

A testator’s will appointed their spouse and sister as joint executors.

The testator’s sister reserved power and a grant of probate was issued to the deceased’s spouse. The testator’s spouse has not yet completed the administration.

Which of the following most accurately describes the sister’s entitlement to act in the administration?

A) The sister can apply for a grant of probate.

B) The sister can act under the original grant provided it mentions that notice of the reservation was given to the testator’s spouse at the time the grant was issued.

C) The sister can apply for a grant of double probate.

D) The sister can act as PR under the original grant issued in the spouse’s name because the sister is named in the will.

E) The sister cannot apply for a grant.

A

C) The sister can apply for a grant of double probate

(Correct. The 2nd grant issued in the estate is a grant of double probate. The other options were incorrect because;
- the new grant was incorrectly stated
-the sister has no entitlement to act under the grant originally issued in the sole name of the spouse
- reserving power does not preclude a later application when the administration is still ongoing
-the original grant will have referred to notice having been given to the sister who reserved power)

81
Q

A deceased’s estate is valued at £200,000 (which includes foreign property worth £75,000). In addition, the deceased is a life tenant of a life interest trust with a capital value at date of death of £100,000. The deceased had never married but had made cash gifts to his siblings in the past seven years totalling £10,000.

Which of the following correctly describes the status of the estate?

A) An exempt excepted estate - the estate is worth less than £650,000.

B) A low value excepted estate - the estate is worth less than £325,000.

C) A low value excepted estate - the estate is worth less than £650,000.

D) An exempt excepted estate - the estate is worth less than £325,000.

E) A non-excepted estate.

A

B) A low value excepted estate - the estate is worth less than £325,000

(Correct. The gross estate is less than the nil rate band, foreign property is less than £100,000, trust property is less than £250,000 and lifetime gifts are less than £250,000. The deceased had never married so no transferrable NRB applies and the relevant NRB amount to consider was £325,000.
The other options were incorrect because no spouse or charity exemption applied (so the estate could not be an exempt excepted estate), the relevant NRB threshold to consider was a single NRB, and the value of the assets described were below the values at which the estate would be prevented from being excepted.)

82
Q

The deceased’s gross estate is valued at £410,000. The estate includes foreign property worth £150,000. The deceased is a life tenant of a life interest trust with a capital value at date of death of £20,000. The deceased had never married and had made no lifetime gifts and left his entire estate to his two children.

Which of the following correctly describes the status of the estate?

A) A low value excepted estate - the estate is worth less than £650,000.

B) A non-excepted estate.

C) A low value excepted estate - the estate is worth less than £325,000.

D) An exempt excepted estate - the estate is worth less than £650,000.

E) An exempt excepted estate - the estate is worth less than £325,000.

A

B) A non-excepted estate

(Correct. The gross value of the estate is above the nil rate band (only one is available) and no spouse or charity exemption applies. IHT is payable and also the value of foreign property is greater than £100,000.
The other options were incorrect because tax was payable on this estate so the estate could not be excepted)

83
Q

The deceased’s gross estate is valued at £300,000. The estate includes foreign property worth £90,000. The deceased is a life tenant of a life interest trust with a capital value at date of death of £270,000. The deceased had never married and had made no lifetime gifts.

Which of the following correctly describes the status of the estate?

A) An exempt excepted estate - the estate is worth less than £650,000.

B) An exempt excepted estate - the estate is worth less than £325,000.

C) A low value excepted estate - the estate is worth less than £325,000.

D) A non-excepted estate.

E) A low value excepted estate - the estate is worth less than £650,000.

A

D) A non-excepted estate

(Correct. Although the gross value of the estate is below the NRB and no IHT is payable, because the value of the trust property is above £250,000 this estate cannot be excepted.
The other options were incorrect as they did not take into account the value of the trust interest)

84
Q

A deceased died without leaving a will. The estate is shared between their spouse and three children (two of whom are minors). No IHT is payable. A paper form will be used for this estate.

Which one of the following statements most accurately describes the requirements when applying for the grant?

A) The PA1A is required and at least two administrators will be appointed under NCPR 22.

B) The PA1P is required and at least two administrators will be appointed under NCPR 22.

C) The PA1A is required and an administrator will be appointed under NCPR 22.

D) The PA1P is required and an administrator will be appointed under NCPR 20.

E) The PA1A is required and at least two administrators will be appointed under NCPR 20.

A

A) The PA1A is required and at least two administrators will be appointed under NCPR 22

(Correct. Where the deceased died without making a will the correct form is the PA1A. Two administrators are required because there will be at minor interests)

85
Q

Which of the following best describes what an applicant under NCPR 22 will always need to do when applying for a grant?

A) Clear-off other applicants

B) Confirm their familiar relationship with the deceased and their beneficial entitlement to the estate

C) Confirm their beneficial entitlement to the estate

D) Confirm their familial relationship with the deceased

E) Confirm the validity of the deceased’s will

A

B) Confirm their familiar relationship with the deceased and their beneficial entitlement to the estate

(Correct. If NCPR 22 applies the deceased died intestate. The entitlement to apply derives from the relationship with the deceased and their right to receive a share of the estate under intestacy, so both must be stated. The other options were incorrect because:
- they were either incomplete or: - It is not always necessary to clear-off anyone else e.g. when the person with the greatest entitlement will be applying. - If NCPR 22 applies the deceased did not leave a valid will)

86
Q

Which of the following is correct with regards an application for a grant of probate?

A) The value of the deceased’s taxable estate will be noted on the grant.

B) The application cannot be made online.

C) The grant can only be issued in the name of an executor as it appears in the will.

D) The application only needs to refer to those executors named in the will who are applying.

E) The grant can be issued in the deceased’s true name and also any common name in which they held assets.

A

E) The grant can be issued in the deceased’s true name and also any common name in which they held assets.

(Correct. The full name of the deceased should be included in the application and where the deceased owned assets in a common name this should also be stated so the grant can be issued in both names. The other options were incorrect because:
- The grant should be issued in the true name of the executors- if this is not how their name appears in the will, the discrepancy can be explained
- The application should refer to all executors named in the will- and if any are not applying the reason for this should be explained
- Most probate applications can be made online
- It is the value of the deceased’s succession (not taxable) estate which appears on the Grant)

87
Q

A testator died leaving a valid will appointing two executors. One of them will renounce. Inheritance tax has been paid. Which of the following most accurately states what needs to be submitted to the probate registry when making an application for the grant?

A) Certified copy of the will and form of renunciation

B) Original will and form of renunciation

C) Certified copy of the will

D) Certified copy of the will and IHT 400

E) Original will, form of renunciation and IHT 400

A

B) Original will and form of renunciation

(Correct. When an executor renounces power they will completed a form of renunciation and this must be submitted along with the application for probate. The original not a copy will must also be provided. The other options were incorrect because: - The original will is needed - The IHT 400 is sent to HMRC not probate registry)

88
Q

An executor takes out a grant of probate and then decides to appoint an attorney to assist with the administration.

Will the power of attorney be sent to the probate registry?

A

No
Correct
Correct. The executor has already taken out the grant in their own name. The probate registry would only be sent the power of attorney if the grant was being issued in the name of the attorney i.e. the executor was delegating their right to apply.

89
Q

A deceased died intestate. The deceased’s surviving spouse inherits the whole of the estate free of inheritance tax (this estate is excepted) and is applying for the grant of representation alone.

Which of the following most accurately states what needs to be submitted to the probate registry when making an application for the grant?

A) Certified copy of the will and IHT 400

B) IHT 400

C) Original will and the probate registry fee

D) The probate registry fee

E) Nothing

A

D) The probate registry fee

(Correct. This is sent to the probate registry as part of the grant application. The other options were incorrect because there was no will (the deceased died intestate) and the IHT 400 is not relevant)

90
Q

A woman dies leaving an estate worth £50,000. By her will the woman leaves £20,000 to her mother and the residue to her sister.

When she died the woman has secured debts of £10,000 and unsecured debts of £20,000.

Which one of the following is correct?

A) The woman’s estate is solvent because there are sufficient funds to pay all of her debts. It does not matter if the legacies cannot be paid in full.

B) The woman’s estate is solvent because there are sufficient funds to pay her secured debts. It does not matter if the unsecured debts or legacies cannot be paid in full.

C) The woman’s estate is solvent because there are sufficient funds to pay her unsecured debts and legacies. The secured debts do not need to be considered as these are tied to specific estate assets.

D) The woman’s estate is insolvent because there are insufficient funds to pay all of her debts and legacies.

E) The woman’s estate is insolvent because after the debts and cash legacy are paid there is nothing left in the residue to pay to her sister.

A

A) The woman’s estate is solvent because there are sufficient funds to pay all of her debts. It does not matter if the legacies cannot be paid in full.

91
Q

A testator dies leaving a will. In his will he leaves his house (worth £600,000), which is subject to a mortgage (£50,000) to his brother, a legacy of £3,000 to his friend and the residue of his estate to his mother. The testator’s assets are worth £50,000 (excluding the house) and he had unsecured debts of £2,000.

The will effectively distributes the whole of T’s estate and is silent with regards the payment of debts.

Which of the following statements is correct?

A) The whole of the estate will bear the burden of the mortgage and unsecured debts and each beneficiary’s entitlement will abate proportionately.

B) The mortgage and unsecured debts will be paid from the sale proceeds of the house.

C) The mortgage and unsecured debts will be repaid from residue.

D) Only the unsecured debts will be repaid from the residue.

D) The unsecured debts will be paid out of the £3,000 set aside for the legacy to T’s friend.

A

D) Only the unsecured debts will be repaid from the residue

(Correct. T’s brother will inherit the house subject to the mortgage (the mortgage will not be repaid using other assets in the estate) – s.35 AEA 1925.
The unsecured debts will be paid in the statutory order prescribed in Sch 1 Part II AEA – the application of which directs the repayment of debts from the residue (as there is no undisposed of part of the estate). The other options were incorrect because:
- The legacy to T’s friend is unaffected
- The house is inherited subject to the mortgage charge and the mortgage debt is not repaid from other estate assets – the value of the house is sufficient to meet the whole of the secured loan
- The house is only subject to the secured debts
- The burden of debts is not shared proportionately between the whole of the estate)

92
Q

The executors of an estate have sought your advice. They need to sell an item from the residuary estate to raise a cash sum of £10,000 to repay a unsecured debt. They have identified the following residuary assets as potentially suitable:

- Painting (probate value £15,000, current value £30,000)

- Car (probate value £10,000, current value £10,000)

- Collection of jewellery (probate value £12,000, current value £12,000)

- Motorbike (probate value £4,000, current value £4,000)

One of the residuary beneficiaries has asked the executors whether they can have the car as part of their entitlement.

Which of the following would be the best advice?

A) It doesn’t matter which item they sell provided the executors fulfil their legal duty to the estate creditors.

B) The executors should sell the motorbike.

C) The executors should sell the painting.

D) The executors should sell the jewellery.

E) The executors should sell the car.

A

D) The executors should sell the jewellery

(Correct. This would be the best option. While the executors have a legal duty to repay the deceased’s debts, and provided they identify items in accordance with the statuary order, they are not legally obliged to sell one item in preference to another, there are practical considerations which they should take into account.
The other items would be less suitable because:
- the painting has increased significantly in value and may result in a capital gains tax charge
- a beneficiary has specifically asked for the car, and although the executors don’t have to agree with the request, in this case there is no reason not to because alternative assets can be sold
- the motorbike would not provide the cash sum that they need.)

93
Q

The deceased acquired an asset when it was worth £10,000. The asset increased in value during the deceased’s lifetime and was worth £40,000 when the deceased died. The PRs needed to sell the asset a few months after the deceased died and have just sold the item for £45,000.

What is the value of the gain made by the PRs?

A) £45,000

B) £5,000

C) None – the gain is treated as being made by the beneficiary who will inherit the asset.

D) £30,000

E) £35,000

A

B) £5,000

(Correct. The PRs are treated as having made a gain equal to the difference in value between the date of death value (£40,000) and the sale proceeds (£45,000).
The other options were incorrect because:
- Gains accrued during the deceased’s lifetime were included
- Sale proceeds are not equivalent to the ‘gain’
- The PRs made the sale not the beneficiary – so the gain is attributed to the PRs)

94
Q

During the administration of an estate the PRs transfer a piano to the deceased’s mother and some shares to the deceased’s brother.

The probate value of the piano was £1,200 and the value at the date of transfer was £2,000. The deceased’s mother has just sold the piano for £1,800.

The probate value of the shares was £3,500 and their value at the date of transfer was £3,000. The deceased’s brother has just sold them for £2,500.

Which of the following is correct?

A) The PRs and the deceased’s mother has made a gain. The deceased’s brother has made a loss.

B) The PRs have made neither a gain nor a loss. The deceased’s mother has made a gain and the deceased’s brother has made a loss.

C) The PRs have made a gain. The deceased’s mother and brother have made a loss.

D) The PRs and both the deceased’s mother and brother have made a loss.

E) The PRs have made neither a gain nor a loss. The deceased’s mother and brother have made a loss.

A

B) The PRs have made neither a gain nor a loss. The deceased’s mother has made a gain and the deceased’s brother has made a loss

(Correct. Where assets are transferred to beneficiaries there is no disposal by the PRs, so they make neither a loss nor a gain. The values at the date of transfer are not relevant.
A beneficiary is deemed to acquire the asset at probate value so the deceased’s:
- mother acquired the piano with a value of £1,200
- brother acquired the shares with a value of £3,500.
When the assets were sold by the beneficiaries, the deceased’s:
- mother sold the piano for £1,800 and would have made a gain of £600
- brother sold the shares for £2,500 and would have made a loss of £1,000)

95
Q

In the tax year of death a testator received rental income of £2,000 in respect of which income tax was due but had not been paid.

During the administration the estate received savings income of £50 as bank interest but no other income.

Which of the following is correct?

A) The PRs are not required to account to HMRC for either the deceased’s rental income or the estate savings income.

B) The PRs should complete a tax return for the deceased for the tax year of death (to include both £2,000 rental income and £50 savings income).

C) The PRs should account to HMRC for tax due in respect of the estate savings income but do not need to worry about the deceased’s untaxed rental income as this does not need to be paid following death.

D) The PRs should complete a tax return for the deceased for the tax year of death (to include £2,000 rental income) and should also account to HMRC for tax due in respect of the estate savings income.

E) The PRs should complete a tax return for the deceased for the tax year of death (to include £2,000 rental income) but do not need to account to HMRC for the estate savings income.

A

E) The PRs should complete a tax return for the deceased for the tax year of death (to include £2,000 rental income) but do not need to account to HMRC for the estate savings income.

(Correct. The deceased’s PR must account to HMRC in respect of the deceased’s untaxed rental income for the tax year of death. They are not however required to account to HMRC for the estate savings income as this was bank interest of under the threshold of £100, and there is no other estate income.)

96
Q

A man died last month leaving a valid will appointing his spouse and brother as executors and giving the whole of his estate to his minor children.

The man’s gross taxable estate was valued at £800,000 and did not include any residential property.

The man’s spouse died two years ago. The man’s brother does has not taken any action so far in the administration of the estate and intends to renounce probate.

Assuming a paper application is required, which of the following most accurately states the items that will comprise the application for the grant of representation?

A) PA1A, IHT 421. Provided the brother has not intermeddled no form of renunciation is required.

B) PA1P.

C) PA1A, IHT 400 and a form of renunciation.

D) PA1P and a form of renunciation.

E) PA1P, IHT 421 and a form of renunciation.

A

D) PA1P and a form of renunciation.

(Correct. The man left a valid will so form PA1P is required, even though the named executors (his spouse and brother) will not be making the application.
Inheritance tax will be payable following the man’s death. The value of his taxable estate is above the NRB and will pass to non-exempt beneficiaries. The man does not have any residential property so the RNRB will not apply. We do not know whether the TNRB may be claimed or not, but even if it did, the value of his estate is greater than 2 x NRB. As such the IHT 400 would be completed and sent to HMRC (not the Probate Registry). HMRC (not the executors) forward an IHT421 to the probate registry.
Renunciation is only possible if the executor has not intermeddled; the man’s brother has not intermeddled and renunciation is possible. A form of renunciation is always required where an executor is renouncing their role)

97
Q

A woman dies leaving a valid will that appoints her spouse as executor and gives all of her assets to him.

The woman’s spouse takes out the grant of probate but he dies before completing the administration of the woman’s estate. He leaves a will appointing their adult son as his executor and the only beneficiary of his will.

Their son has just obtained the grant of probate for his father’s estate.

Which of the following is correct regarding who is able to administer the woman’s estate?

A) The son is not able to complete the administration of his mother’s estate because he is not named as her executor.

B) The son is not able to complete the administration of his mother’s estate without making a further application to the probate registry.

C) The son is able to complete the administration of his mother’s estate under the authority conferred by the grant of probate of his father’s estate.

D) The son is able to complete the administration of his mother’s estate because he is the ultimate beneficiary of her estate.

E) The son is able to complete the administration of his mother’s estate because he is the sole beneficiary of his father’s estate.

A

C) The son is able to complete the administration of his mother’s estate under the authority conferred by the grant of probate of his father’s estate.

(Correct. The chain of representation under s.7 Administration of Estates Act 1925 applies. As the woman’s spouse had taken out the grant of probate of her estate before he died, and their son had taken out the grant of probate of his father’s estate, the chain of representation is complete. The son can complete the administration of both estates under authority conferred by the grant in his father’s estate. No further application is required)

98
Q

A woman left a valid will which appoints her civil partner as her executor and gives the whole of the woman’s estate to her adult sister.

The woman was survived by her mother, sister and adult son. The woman’s civil partner died before the woman.

Which of the following correctly states who would apply for the grant of representation of the woman’s estate?

A) The woman’s sister and mother (two administrators are required)

B) The woman’s sister

C) The woman’s son

D) The woman’s mother

E) The personal representative of the woman’s civil partner (under the chain of representation)

A

B) The woman’s sister

(Correct. The woman left a will which does not validly appoint any executors. Therefore NCPR 20 applies.
Under NCPR 20 the person with the greatest entitlement to apply is the beneficiary of the woman’s residuary estate, here her sister.
The other options were incorrect because neither the mother nor son are beneficiaries and do not rank in priority above the sister. Only one PR is required (there are no life or minor interests) and the chain of representation does not apply)

99
Q

A woman died leaving a valid will which shares her estate equally between her adult children. The woman’s sister has been appointed as executor of the woman’s estate.

One of the woman’s three children cannot be located and has been missing since going travelling two years ago. Despite carrying out extensive searches no one can find the missing beneficiary.

The woman’s sister wants to complete the administration of the woman’s estate and distribute the assets between the two other children. However, she is concerned about her personal liability should the missing child return later and claim their 1/3 share.

Which one of the following would not adequately protect the woman’s sister?

A) Making a payment into court of the missing beneficiary’s 1/3 share.

B) Retaining the missing beneficiary’s 1/3 share and delaying the administration of the woman’s estate.

C) Distributing the estate to the two other children and obtaining insurance to protect against a subsequent claim by the missing beneficiary.

D) Obtaining a Benjamin Order.

E) Placing a notice in accordance with s.27 Trustee Act 1925.

A

E) Placing a notice in accordance with s.27 Trustee Act 1925.

(Correct. All of the other options are feasible where there is a known but missing beneficiary. The protection afforded by a s.27 Trustee Act 1925 notice is in relation to unknown creditors and beneficiaries, not those who are known about but cannot be located.)

100
Q

A woman makes a PET with a chargeable value of £50,000. She does the same 11 years later. A year after the second gift, she dies. She makes no other lifetime gifts. What is the cumulative total on her death?

£50,000

£175,000

£100,000

£325,000

£0

A

£50,000

Correct: The woman has made one PET of £50,000 in the last seven years.

101
Q

Which one of the following scenarios does not include a chargeable event for inheritance tax purposes?

A PET which is made 9 years before death

An LCT which is made 15 years before death

A PET which is made 3 years before death

An LCT and a PET made together

The death of an individual who has made no lifetime gifts

A

A PET which is made 9 years before death

Correct: A PET made more than 7 years before death is not chargeable.

102
Q

A man gifts £500,000 into trust. If the man lives for another ten years, which one of the following is correct in relation to the inheritance tax treatment of the gift?

Tax will be payable at the time of transfer at the death rate of 40%

No tax will be payable at the time of the transfer

Tax will be payable at the lifetime rate of 20%

Tax will initially be payable at the lifetime rate of 20% then reassessed at the death rate of 40%

Tax will be payable on the man’s death at the death rate of 40%

A

Tax will be payable at the lifetime rate of 20%

Correct: The gift is an LCT. The man survives more than seven years after the LCT so it is not reassessed at the death rate.

103
Q

A man makes three gifts in his lifetime, each with a chargeable value of £10,000. The first gift is made 15 years before his death, the second is five years before his death and the final one is gifted in the week of his death. What is the basic Nil Rate Band available to his death estate? (There is no TNRB available.)

£30,000

£295,000

£325,000

£20,000

£305,000

A

£305,000

Correct: Two of the gifts were made in the seven years before the man’s death so are included in the cumulative total.

104
Q

A woman died two years ago and left her entire estate worth £400,000 to her husband. She made no lifetime gifts. Her husband died recently. He left his entire estate to his children. His estate was valued at £900,000 including a family home worth £500,000. He made three PETS in his lifetime totalling £500,000. All three were made over 20 years ago.

Which one of the following is correct in terms of available Nil Rate Bands?

His estate can claim a basic NRB and his wife’s basic NRB only

His estate will have no NRB available to it

His estate can claim a basic NRB only

His estate can claim a basic NRB, his wife’s basic NRB, his RNRB and his wife’s RNRB

A

His estate can claim a basic NRB, his wife’s basic NRB, his RNRB and his wife’s RNRB

Correct: The man and his wife both died with full basic NRB available (his PETs were made more than 7 years ago so the man’s cumulative total when he died was zero). The wife also did not use her RNRB so this is also available to transfer.

105
Q

What is the maximum combined value of the nil rate bands available to the estate of a surviving spouse?

£825,000

£500,000

£650,000

It depends how many times the deceased was married as they can receive TNRB from each former spouse

£1,000,000

A

£1,000,000

Correct: A surviving spouse can make use of their own basic NRB (£325,000) and RNRB (£175,000) and receive a transfer of up to 100% of the basic NRB and RNRB from the estate of their deceased spouse.

106
Q

A man makes an LCT (‘the LCT’). He has previously made the following transfers:

An LCT (chargeable value £50,000) 9 years before the LCT

A PET (if failed, chargeable value would be £10,000) 8 years before the LCT

A PET (if failed, chargeable value would be £15,000) 6 years before the LCT

An LCT (chargeable value £25,000) 4 years before the LCT

What is the cumulative total that applies when calculating the IHT due at the lifetime rate on the recent LCT?

£75,000

£15,000

£100,000

£25,000

£40,000

A

£25,000

Correct: The £25,000 LCT is included in the cumulative total as it was made within the 7 years prior to the LCT. The other options were wrong because neither PET has yet failed (so are not included within the cumulative total) and the first LCT was made more than 7 years before the recent LCT.

107
Q

A woman settles £650,000 on trust for her nieces and nephews (an LCT). She has made no previous lifetime transfers.

Calculate the IHT due on the LCT at the time it is made.

£65,000

£63,800

£127,000

£130,000

£64,400

A

£63,800

Correct: The woman has a cumulative total of £0 (Step A) and is able to use her AE from the year of the LCT and the previous year to reduce the value of the LCT to £644,000 (Step C). The NRB is deducted from the chargeable value, leaving £319,000 to be taxed at the lifetime rate of 20% Step D). The other answers were wrong because they missed one or both lots of AE and/or applied IHT at the death rate of 40%.

108
Q

A woman settles £650,000 on trust for her nieces and nephews (an LCT). She has made no previous lifetime transfers.

Calculate the IHT due on the LCT at the time it is made.

£63,800

£64,400

£130,000

£65,000

£127,000

A

£63,800

Correct: The woman has a cumulative total of £0 (Step A) and is able to use her AE from the year of the LCT and the previous year to reduce the value of the LCT to £644,000 (Step C). The NRB is deducted from the chargeable value, leaving £319,000 to be taxed at the lifetime rate of 20% Step D). The other answers were wrong because they missed one or both lots of AE and/or applied IHT at the death rate of 40%.

109
Q

An unmarried man died recently. The only lifetime transfer he made was an LCT (value £550,000) four and a half years before he died. IHT of £43,800 was paid on the LCT at the time of the transfer.

Calculate the IHT due on the LCT at the date of the man’s death.

£43,800

£10,200

£8,760

£52,560

£87,600

A

£8,760

Correct: The man had a cumulative total of £0 (Step A) and was able to use his AE from the year of the LCT and the previous year to reduce the chargeable value to £544,000 (Step C). The NRB is deducted from the chargeable value, leaving £219,000 to be taxed at the death rate of 40% (Step D). As the LCT was 4 years before death, taper relief is applied at 40% (Step E) and then credit is given for the IHT paid during his lifetime (Step F). The other answers were wrong because they failed to apply one or more of the AE at Step C, taper relief at Step E or credit for the lifetime tax at Step F.

110
Q

A woman died six months ago. Her estate now includes the following debts:

i. Legal fees for probate work ii. Income tax bill for period up to death iii. Credit card bill for pre-death spending iv. Funeral bill v. Post-death house clearance invoice

Which of the following correctly lists the debts that can be deducted for inheritance tax purposes?

None of them

The income tax, credit card bill and funeral bill

All of them

The funeral bill only

The legal fees, income tax, credit card bill and funeral bill

A

The income tax, credit card bill and funeral bill

Correct: The deceased’s income tax and pre-death liabilities are deductible, as are funeral expenses. The other expenses are not deductible for tax purposes (but can be paid from the estate assets).

111
Q

A solicitor has calculated the inheritance tax due on his client’s estate. The client had never married and left her entire estate to her brother. The estate consisted of personal chattels, cash in a bank account and a house owned as joint tenants with her brother.

The solicitor has reached the wrong figure for the inheritance tax due.

Which of the following has caused the error?

When calculating the tax due, the solicitor skipped step 6 of the IHT calculation and went straight to step 7.

When valuing the taxable death estate, the solicitor did not include the deceased’s share of a house held as joint tenants with her brother because joint tenancy assets pass via survivorship.

When calculating the tax due at step 7 of the IHT calculation, the solicitor only applied the basic NRB and did not consider the availability of the transferrable NRB.

When calculating the value of the taxable estate the solicitor deducted funeral expenses.

When calculating the cumulative total, the solicitor included three LCTs (made 4, 5 and 6 years ago) but did not include a PET (made 8 years ago).
Correct
Correct: Although assets owned as joint tenants pass via survivorship (and therefore do not form part of the succession estate) the deceased’s interest is still part of the taxable death estate. By failing to include the value of the deceased’s share of the house, the solicitor will have reached the wrong figure for the taxable death estate.

A

When valuing the taxable death estate, the solicitor did not include the deceased’s share of a house held as joint tenants with her brother because joint tenancy assets pass via survivorship.

112
Q

Death estate 7 step calculation?

A) Step 1 - Identify the taxable estate Step 2 - Calculate the cumulative total Step 3 - Value the taxable estate Step 4 - Deduct debts Step 5 - Apply exemptions and reliefs Step 6 - Apply RNRB Step 7- Apply NRB and calculate tax

B) Step 1 - Calculate the cumulative total Step 2 - Identify the taxable estate Step 3 - Value the taxable estate Step 4 - Deduct debts Step 5 - Apply exemptions and reliefs Step 6 - Apply NRB Step 7- Apply RNRB and calculate tax

C) Step 1 - Calculate the cumulative total Step 2 - Identify the taxable estate Step 3 - Value the taxable estate Step 4 - Apply the RNRB and NRB Step 5 - Deduct debts Step 6 - Apply exemptions and reliefs Step 7- Calculate tax

D) Step 1 - Calculate the cumulative total, Step 2 - Identify the taxable estate, Step 3 - Value the taxable estate, Step 4 - Deduct debts, Step 5 -Apply exemptions and reliefs, Step 6 - Apply RNRB, Step 7 - Apply NRB and calculate tax

E) Step 1 - Identify the taxable estate Step 2 - Calculate the cumulative total Step 3 - Value the taxable estate Step 4 - Apply the RNRB and NRB Step 5 - Deduct debts Step 6 - Apply exemptions and reliefs Step 7- Calculate tax

A

D) Step 1 - Calculate the cumulative total, Step 2 - Identify the taxable estate, Step 3 - Value the taxable estate, Step 4 - Deduct debts, Step 5 -Apply exemptions and reliefs, Step 6 - Apply RNRB, Step 7 - Apply NRB and calculate tax

Correct. It is necessary to start by calculating the cumulative total, then identify and value the taxable estate. Deduct debts from the gross value of the estate to reach the net value, then apply exemptions and reliefs. RNRB should be applied before the basic NRB, then calculate the tax.

113
Q

A woman died leaving a will that appoints her daughter as executrix. There are no provisions within the will which affect the incidence of payment of inheritance tax (‘IHT’).

The woman’s taxable estate contains the following items:

Bank accounts

Property owned tenants in common with her mother

A life interest in a trust

Who will be liable for paying IHT following the woman’s death?

The woman’s mother and trustees of the trust

The woman’s daughter and mother

The woman’s daughter, mother and trustees of the trust

The woman’s daughter and the trustees of the trust

The woman’s daughter only

A

The woman’s daughter and the trustees of the trust

Correct: The liability for paying tax on the free-estate (here the bank accounts and share of the property owned as tenants in common) lies with the woman’s daughter as her executor under the general rule. The liability for paying tax on the trust interest lies with the trustees of the trust.

114
Q

A man dies leaving a will that makes no express provision relating to the payment of inheritance tax (‘IHT’). By his will the man gives his:

Quoted shares to his father (no business property relief applies) (£40,000)

Personal possessions to his mother (£30,000)

£50,000 to his niece (this gift is not expressed to be free of tax)

The remaining assets in his estate are worth £220,000 and pass to the man’s nephew under the gift of residue.

Which of three assets noted above should the man’s PRs use to pay the IHT due following his death?

Personal possessions

Any of them – the PRs may choose which items are used to meet general testamentary expenses

£10,000

None of them

Quoted shares

A

None of them

Correct: The general rule is that the residuary estate bears the burden of the IHT due in respect of the assets passing to the PRs and that other gifts in the will are made free of tax.
The other options were incorrect because they involved using assets given away specifically in the will and did not form part of the residuary estate.

115
Q

A woman dies 9 years after making an LCT and 2 years after making a large PET to her daughter.

Inheritance tax is payable following her death estate.

By her will the woman appoints her spouse as her executor, gives her house to her brother and leaves the remainder of her estate “after all taxes have been paid” to her grandson.

Which of the following is correct with regards the payment of IHT due following the woman’s death?

The trust assets cannot be used to pay any of the IHT due

The woman’s daughter cannot be liable to pay IHT

The burden of the IHT will be shared between the woman’s brother and grandson

Any IHT payable in respect of the lifetime transfers will be met from the residuary estate (the general rule has been varied)

The value of the LCT must be grossed-up before the IHT liability can be determined

A

The trust assets cannot be used to pay any of the IHT due

Correct: The LCT was made more than 7 years before her death and is therefore not assessed to IHT at this time. As such, none of the trust assets can be used to meet the IHT bill. The other options were incorrect because:
The PET failed and if IHT is due in respect of the lifetime transfer, the woman’s daughter would be liable to pay this.
Grossing up occurs in respect of the IHT due on creation of the trust, not following death.
The burden of IHT would not be shared between the beneficiary of a specific gift and the residuary beneficiary unless the testator had given a direction in her will to vary the general rule.
The expression “all taxes” does not vary the general rule.

116
Q

True or False: IHT exemptions can only be used to reduce the IHT liability on a person’s death estate, or in respect of lifetimes transfers, but not both.

True

False

A

False

Correct: There are exemptions and reliefs that apply to both lifetime transfers and the death estate.

117
Q

True or False: The basis of most IHT exemptions and reliefs is common law.

False

True

A

False

Correct: Exemptions and reliefs are available by virtue of statutory authority and are set out in the Inheritance Tax Act 1984. Case law relating to exemptions and reliefs usually concerns a dispute over the application of statutory rules.

118
Q

True or False: The nil rate band (‘NRB’) is not an IHT exemption or relief.

False

True
Correct
Correct: The NRB is a 0% rate of tax. To the extent the value of a transfer falls within the NRB there is no tax to pay, which is why people sometimes confuse the NRB as an exemption/relief.

A

True

Correct: The NRB is a 0% rate of tax. To the extent the value of a transfer falls within the NRB there is no tax to pay, which is why people sometimes confuse the NRB as an exemption/relief.

119
Q

A woman made a gift of £500,000 to her daughter 3.5 years before she died.

Which of the following is correct in relation to the woman’s lifetime gift?

The gift is chargeable to IHT following the woman’s death. Taper relief applies and only 20% of the IHT due is now payable.

The gift is a PET which means it is not subject to IHT and therefore taper relief is irrelevant.

The gift is chargeable to IHT following the woman’s death. Taper relief applies and a 20% reduction in the IHT due can be claimed.

The gift is chargeable to IHT following the woman’s death. Taper relief applies and an 80% reduction in the IHT due can be claimed.

The gift is a PET, which means it was not chargeable when it was made. Therefore, there is no IHT liability which can be tapered.

A

The gift is chargeable to IHT following the woman’s death. Taper relief applies and a 20% reduction in the IHT due can be claimed

Correct: The woman has made a gift to another person during her lifetime (a PET) and then died within 7 years after making the gift (the PET has failed). The failed PET is chargeable following the woman’s death. As the woman survived more than 3 years after making the gift, taper relief will apply. The rate of taper is a 20% reduction in the IHT due (i.e. 80% of the IHT due is payable).
The other options were incorrect because:
-The PET is chargeable - it failed as the woman died less than 7 years after making the gift
- Taper relief applies as the woman survived more than three years after making the gift
- The rate of taper is 20% and therefore 80% remains payable

120
Q

A couple are getting married. What is the maximum amount of IHT marriage relief that can be claimed in relation to a gift to the couple by the groom’s aunt?

£5,000

£2,500

None

£1,000

£2,000

A

£1,000

Correct. The aunt is not a parent or grandparent so the other figures do not apply. She cannot give £1,000 to each party - the relief applies “per marriage”

121
Q

A man makes a gift into trust (LCT) worth £15,000. The man has made no other lifetime transfers.

Which one the following exemptions/reliefs applies?

Small gifts

Annual exemption of £3,000

Woodlands relief

Marriage

Annual exemption of £6,000

A

Annual exemption of £6,000

Correct: The man can claim the annual exemption (‘AE’) of £3,000 for the tax year of the gift. As he has made no other lifetime transfers we know he has not used his AE from the previous year, so this may be claimed as well, which gives a total of £6,000.
The other options were incorrect because:
AE of more than £3,000 can be claimed
Small gifts allowance cannot apply to a gift worth more than £250, nor to a gift into a trust
For marriage exemption to apply the gift must be made to a party of the marriage (not a trust)
Woodlands relief is not available in respect of lifetime transfers.

122
Q

A man wants to give away some of his assets to his civil partner. Both parties are domiciled in the UK.

The man is considering making a gift of cash now and then giving the remainder of his estate to his civil partner by his will to take effect on the man’s death.

Which of the following represents the most accurate advice for the man?

The gift of cash and the gift of the estate will be exempt from IHT irrespective of their value.

The gift of cash will not be exempt from IHT. The gift of the man’s estate will not be exempt from IHT if it is conditional upon the man’s civil partner surviving him by 28 days.

The gift of cash and the gift of the estate will be exempt from IHT up to the value of the man’s nil rate band.

The gift of cash will be exempt from IHT provided the man survives 7 years after making the transfer.

The gift of cash will be exempt from IHT. The gift of the man’s estate cannot be exempt from IHT if the man choses to leave his assets into a life interest trust.

A

The gift of cash and the gift of the estate will be exempt from IHT irrespective of their value.

Correct: Spouse/civil partner exemption applies to lifetime transfers and to gifts made under a will – any assets passing to the man’s civil partner will be entirely free of IHT.
The other options were incorrect because:
There is no upper limit to the amount of the spouse exemption where both parties are UK domiciled.
Although the gift of cash to a person during the man’s lifetime would usually be a PET, this cash transfer is exempt by virtue of spouse/civil partner exemption and it does not matter when the man dies
The man could leave his assets into a life interest trust and spouse exemption could still apply provided the man’s civil partner is the life tenant
A standard survivorship clause in a will does not prevent the application of spouse/civil partner exemption

123
Q

A woman’s will contains the following clause:

“I give £8,000 to Macmillan Cancer Support (registered charity number 261017) of [ address ] for its general purposes”

The woman has a full nil rate band (NRB) available.

How much IHT is payable on this gift?

£3,200 (£8,000 x 40%) – if her estate is valued above the NRB.

£0 – because the value of the gift is below the NRB.

£2,880 (£8,000 x 36%) – if her estate is valued above the NRB.

£0 – if the value of her estate is below the NRB.

£0 – irrespective of the value of her estate.

A

£0 – irrespective of the value of her estate.

Correct: Charity exemption applies and this gift is fully exempt. The size of the gift and the value of the other assets in her estate are irrelevant. The reduced death rate of IHT at 36% is only ever applied to the chargeable assets within an estate – it is not a rate of tax applied to a charitable gift itself.

124
Q

True or false: All of the following are available to use in respect of both lifetime gifts and the death estate

  • Political party exemption

-Exemptions for gifts for national purposes or to heritage maintenance funds

-Exemption for gifts to EBTs

-Exemption for gifts to housing associations

True

False

A

True

Correct: All of these are available to use in respect of lifetime transfers and the death estate. You are not required to know about these exemptions in detail.

125
Q

A man entered into a business partnership with a friend. They agreed to hold the shares in their new venture on a 50:50 basis. The man died six months later.

Based on the information provided, which one of the following is correct in relation to the inheritance tax position following the man’s death?

APR will be available at 50%

APR will be available at 100%

BPR will be available at 100%

BPR will be available at 50%

No IHT reliefs will be available

A

No IHT reliefs will be available

Correct: The qualifying period of ownership has not been satisfied for either BPR or APR so neither relief is relevant.

126
Q

A woman owns shares in a trading company called “It’s all going to be OK Ltd”. The woman purchased the shares 2.5 years ago and they are now worth £100,000. The woman does not have a controlling interest.

The woman has just died and leaves all of her taxable estate to her daughter.

Which one of the following is correct with regards business property relief?

BPR of £50,000 can be claimed

No BPR can be claimed as she did not have a controlling interest

BPR can only be claimed in respect of the woman’s estate if her daughter keeps the shares for more than 2 years

No BPR can be claimed as she did not survive more than 3 years after having acquired the shares

BPR at £100,000 can be claimed

A

BPR at £100,000 can be claimed
Correct

Correct. The woman owned shares in a private company (it ends with “ltd”) for more than 2 years before she died. BPR applies at 100%, ie £100,000.
The other options were incorrect because:
The 50% rate is not applicable (these are not quoted shares)
There is no requirement to have a controlling interest for a private company shareholding to qualify
The qualifying period of ownership is 2 years, not 3
The period of time her daughter owns the shares for is irrelevant when considering the availability of BPR for the woman’s estate

127
Q

An unmarried woman dies owning land at an agricultural value of £300,000. The open market value of the land is £360,000. She has farmed the land herself for 5 years.

Which one of the following is correct in relation to her death estate?

If the land qualifies for BPR, this will be claimed instead of APR.

Neither APR nor BPR can be claimed.

APR worth £300,000 can be claimed.

APR worth £360,000 can be claimed.

APR worth £150,000 can be claimed.

A

APR worth £300,000 can be claimed

Correct: The woman has owned and occupied the land for the purposes of agriculture for more than 2 years. The conditions of APR are met. APR can be claimed at 100% of the agricultural (not open market) value of the land. The other options are incorrect because: APR applies at 100% in this case, not 50% APR applies to the agricultural value, not market value, of the land Where both APR and BPR apply, APR takes priority.

128
Q

A woman dies leaving a will that gives all of her estate to her sister. The woman and her sister were joint tenants of a property. The woman’s estate was taxable.

The woman’s sister dies 1 year later leaving her own estate (which includes assets inherited from the woman) to her children. The sister’s estate was not subject to IHT.

Which of the following is correct with regards quick succession relief (‘QSR’)?

QSR would apply to both the woman and the sister’s estate.

QSR would apply to the sister’s estate only.

QSR would apply to the woman’s estate only.

QSR would apply to the woman’s estate but cannot apply to the value of the property as this passed automatically to the sister by survivorship.

QSR does not apply to either estate.

A

QSR does not apply to either estate.

Correct. For QSR to apply both relevant estates must have been taxable –here the sister’s estate was not subject to IHT. The other options were incorrect as QSR does not apply.

129
Q

A man dies and his estate comprises cash, a partnership interest, and a car (which he inherited from his brother 6 months ago). The man leaves a will that includes cash gifts to his spouse (£10,000) and a local registered charity (£2,000).

Which one of the following exemptions cannot be considered when calculating the IHT liability in respect of the man’s death estate?

Annual exemption

Charity exemption

Quick succession relief

Business property relief

Spouse exemption

A

Annual exemption

Correct. The annual exemption cannot be claimed in respect of a person’s death estate. The other exemptions and reliefs were all potentially relevant:
BPR in respect of the partnership interest
Spouse exemption in respect of the £10,000 gift in his will
Charity exemption in respect of the £2,000 gift in his will
Quick succession relief in respect of the car he inherited from his brother

130
Q

A man dies leaving the whole of his estate to his son. No inheritance tax is payable following his death.

The man’s estate comprises only his bank accounts (£4,000), a house and its surrounding woodland (combined value of £250,000) and his personal possession (£1,500). The man purchased the house and woodland six years ago. The timber value of the land is £15,000.

Which of one of the following is correct?

A claim for woodlands relief can be made (with reference to the value of £15,000).

A claim for woodlands relief would be made when the man’s son sells the land.

A claim for woodlands relief can be made (with reference to the value of £250,000).

It would be preferable for the man’s estate to make use of BPR instead of woodlands relief.

Woodlands relief is not applicable.

A

Woodlands relief is not applicable.

Correct. Woodlands relief is a deferral of inheritance tax (‘IHT’) that would otherwise be payable. This is estate is not subject to IHT so there is nothing to defer.

131
Q

A client comes to see you to discuss IHT planning. The client’s estate is taxable. The client is not married and is not close with any of their family aside from their only child (21) who is studying full time at university. The client spends roughly 90% of their income each year, leaving 10% unused. The client has savings of about £60,000.

On these facts, which of the following IHT exemptions are likely to be the most useful for this client?

Family maintenance, annual exemption and normal expenditure from income.

Spouse exemption, small gifts allowance and normal expenditure from income.

Marriage exemption, family maintenance and the annual exemption.

Family maintenance and spouse exemption.

Annual exemption and small gifts allowance.

A

Family maintenance, annual exemption and normal expenditure from income.

Correct. The client could maintain their adult child while in full time education and make use of the family maintenance exemption. The client could also make regular gifts of their spare income and claim normal expenditure from income. The annual exemption would be useful for gifts that would not otherwise be covered by another exemption. The other options were wrong because:
The client was not married
There was nothing to suggest that the client would make multiple gifts of small amounts to other family members
There is no marriage taking place

132
Q

The client owns a valuable family home, a rental property and significant cash savings.

Which of the following represents poor IHT planning advice?

The client could take out a life insurance policy and write the benefit of the policy in trust for his children.

The client could ensure his spouse has sufficient cash to give away £3,000 each tax year.

The client could give away £3,000 each tax year.

The client could transfer the rental property to his children and pay them rent for any use he later makes of the property.

The client could transfer the family home to his adult children and continue to live there rent free provided the children agree.

A

The client could transfer the family home to his adult children and continue to live there rent free provided the children agree.

Correct – this option would not be good IHT planning. If a person gives away an asset that they continue to benefit from they make a gift with reservation of benefit (GROB). For IHT purposes the client would continue to be treated as the owner of the property and their estate for IHT purposes would be no smaller than if they had simply done nothing.

133
Q

A client comes to see you to discuss IHT planning as they have just won the sum of £400,000 from playing the national lottery. The client can afford to give away most of this amount. There is no specific person that needs a large sum of money up front but the client would like to benefit a number of different family members.

Is the following true or false?

The client should make a single PET to the value of £400,000 and purchase insurance to cover the risk that IHT becomes payable if the client dies in the 7 years that follow the gift.

True

False

A

False

Correct. A single PET would be a gift to one person which the client has indicated is not suitable. It would be better to set up a trust to benefit a wider group of people i.e. make an LCT instead of a PET.

134
Q

Tax avoidance is a criminal offence.

True or false?

True

False

A

False

Correct: Tax evasion is a criminal offence. Tax avoidance, however aggressive, is not.

135
Q

A woman dies. Her estate consists of her house (valued at £500,000), £200,000 in a bank account and shares worth £100,000 (which qualify for BPR at 100%). She acquired the shares with a loan of £100,000. There is £50,000 outstanding on the loan at the date of her death. Her funeral expenses are £10,000.

What is the value of the woman’s taxable estate once debts and reliefs have been deducted?

£690,000

£700,000

£640,000

£600,000

£590,000

A

£690,000

Correct: The woman’s assets are worth £800,000. The funeral expenses and loan are deducted to reach a figure of £740,000. As the loan is deducted from the value of the shares (reducing them to £50,000) only £50,000 can be deducted for BPR. This results in a figure of £690,000.

136
Q

A woman buys a holiday cottage in Cornwall (England) for £200,000. She stays in the cottage for several weeks every year and rents it out to holidaymakers for the rest of the year. Eight years before the woman dies, she makes a gift of the cottage to her daughter. At the time of the gift, the cottage is worth £300,000. The woman continues to stay in the cottage for several weeks every year and receives the rental income until her death. At the date of the woman’s death, the cottage is valued at £500,000.

How is the cottage treated for inheritance tax (‘IHT’) and capital gains tax (‘CGT’) purposes in the hands of the woman and her daughter respectively?

For IHT purposes, the disposal of the cottage is a potentially exempt transfer which takes place more than seven years before the woman’s death and is therefore not taxable. For CGT purposes the cottage is treated as passing on the woman’s death, meaning no CGT is payable on the transfer and the woman’s daughter acquires it at its market value of £500,000.

For IHT purposes, the cottage is treated as part of the woman’s taxable estate and is valued at £500,000. For CGT purposes, the woman disposes of the cottage for £300,000, resulting in a gain of £100,000. Her daughter acquires the cottage at its market value of £300,000.

For IHT purposes, the cottage is treated as part of the woman’s taxable estate and is valued at £500,000. For CGT purposes the cottage is also treated as passing on the woman’s death, meaning no CGT is payable on the transfer and the woman’s daughter acquires it at its market value of £500,000.

For inheritance tax purposes, the cottage is treated as part of the woman’s taxable estate and is valued at £300,000. For capital gains tax purposes, the woman disposes of the cottage for £300,000, resulting in a gain of £100,000. Her daughter acquires the cottage at its market value of £300,000

For IHT purposes, the disposal of the cottage is a potentially exempt transfer which takes place more than seven years before the woman’s death and is therefore not taxable. For capital gains tax purposes, the woman disposes of the cottage for £300,000, resulting in a gain of £100,000. Her daughter acquires the cottage at its market value of £300,000.

A

For IHT purposes, the cottage is treated as part of the woman’s taxable estate and is valued at £500,000. For CGT purposes, the woman disposes of the cottage for £300,000, resulting in a gain of £100,000. Her daughter acquires the cottage at its market value of £300,000.

Correct: The gift is a GROB which does not cease before the woman’s death so it is treated as part of her taxable estate for IHT purposes. It is also a potentially exempt transfer but it is not reassessed because it was made more than 7 years before the woman’s death, so it is not reassessed when the woman dies. The lifetime gift is treated as a disposal for CGT purposes, meaning the woman has made a gain (which will be chargeable because it’s not her main residence) and her daughter does not benefit from the free CGT uplift

137
Q

Each of the following testators made a will leaving their whole estate to their surviving spouse.

Testator A is married and owns the following: a rental property worth £350,000, cash savings of £40,000 and personal possessions worth £15,000. Testator A has a full nil rate band available.

Testator B is married and owns the following: a residential property owned jointly with their spouse which is worth £250,000, cash savings of £7,000 and personal possessions worth £10,000. Testator B used part of their nil rate band and only £280,000 remains.

Testator C was married when their will was drafted but subsequently divorced. Testator C owns property valued at £500,000, has investments worth £30,000 and personal possessions worth £25,000. Testator C has no available nil rate band.

For which of these testators will spouse exemption reduce or eliminate the charge to inheritance tax following their death?

Testator A only.

All of them.

Testator B only.

Testators B and C only.

Testators A and B only.

A

Testator A only

Correct. Spouse exemption is useful where the deceased’s estate would otherwise be taxable. Testator A has assets worth more than the nil rate band and therefore IHT would be payable if spouse exemption did not apply.
The other options were incorrect because:
Testator B does not have a taxable estate. The value of the assets is less than the nil rate band so no tax saving is achieved.
Testator C is not married when they died so spouse exemption does not apply.

138
Q

A client comes to see you for advice on making a new will. The client wants to share their assets between their nieces and nephews and a local charity. The clients owns property worth £210,000, has cash savings of £100,000 and personal possessions worth £30,000. The client has a full nil rate band available. You may assume there are no estate liabilities.

The client proposes making a cash gift to the charity of £34,000.

Which of the following is correct?

The gift to charity is exempt and the remainder of his estate would be taxed at 40%.

No IHT would be payable on the man’s estate.

All of the estate will be taxed at 36%.

The gift to charity is exempt and the remainder of the estate would be taxed at 36%.

The gift to charity is taxed at 36% and the remainder of the estate is taxed at 40%.

A

No IHT would be payable on the man’s estate

Correct. The man’s estate is worth £340,000. The gift to charity of £34,000 qualifies for 100% relief. The remainder of his estate worth £306,000 is within the nil rate band so no tax is due.
The other options were incorrect because:
Although the charitable gift is worth at least 10% of the estate, after the nil rate band is applied, there is no part of the estate chargeable to tax to which the reduced rate can apply.
If tax were payable, the reduced rate of IHT would apply.
The reduced rate of tax does not apply to charitable gift

139
Q

A client owns shares in a ltd company set up by their friend. The shares are currently qualifying business assets and are worth £20,000. The testator’s other assets comprise his savings (£300,000), car (£15,0000), personal possessions (£50,000).

The client wishes to make a will that provides for his daughter and leaves the residue of his estate to his civil partner.

The client has no nil rate band available.

If the client wishes to make efficient use of business property relief (BPR), which of the following would be appropriate advice with regards their will?

The testator should give the shares to his civil partner who will then claim BPR when they die.

If the testator is worried that the shares might fall in value he should sell them and leave a specific cash legacy of the sale proceeds to his daughter.

The testator should ensure the shares form part of the residue of his estate.

For BPR to apply to the legacy the shares must qualify on the date the client signs their will.

The testator could make a specific gift of the shares to his daughter.

A

The testator could make a specific gift of the shares to his daughter.

Correct. The client’s estate is taxable to the extent it does not pass to his civil partner. As such, it is efficient to give exempt assets to the chargeable beneficiary. If the shares were given to the client’s civil partner BPR would be lost. The other options were incorrect because:
- The shares must qualify for BPR when the testator dies (not executes his will).
- It is not efficient to make a specific gift of exempt assets to an exempt beneficiary, and there is no guarantee the assets will still qualify for BPR on the death of the civil partner.
- It is not efficient for the exempt assets to form part of residue where the residue is given to an exempt beneficiary.
- A gift of cash will not qualify for BPR.

140
Q

For which one of the following taxable estates (all worth £3m) would the inheritance tax be calculated with reference to the total value of the residuary estate?

A will gives “£400,000 to my spouse and the residue of my estate equally between my spouse and my son”

A will gives “£400,000 to my daughter and the residue of my estate to my spouse”

A will gives “£400,000 to my spouse and the residue of my estate to my son”

A will gives “£400,000 to my daughter and the residue of my estate to my son”

A will gives “£400,000 to my daughter and the residue of my estate equally between my spouse and my son”

A

A will gives “£400,000 to my spouse and the residue of my estate to my son”

Correct. The benefit of spouse exemption applies to the specific legacy so this amount is disregarded. The IHT due is therefore calculated with reference to the assets passing under the residue to the testator’s son. The other options were incorrect because they either included a specific gift to a chargeable beneficiary (i.e. the sum of £400,000 would not be excluded before the tax was calculated) or the residue was partially exempt so its full value would not be subject to IHT.

141
Q

For which one of the following estates would the legacy of £50,000 require grossing-up before any inheritance tax is calculated?

Assume that no nil rate band is available following the death of each testator.

A will gives “£50,000 subject to tax to my son and the residue of my estate to my wife”

A will gives “£50,000 subject to tax to my wife, and the residue of my estate to my son”

A will gives “£50,000 free of tax to my son and the residue of my estate to my wife”

A will gives “£50,000 to my wife, and the residue of my estate to [ABC registered charity]”

A will gives “£50,000 free of tax to my wife, and the residue of my estate to my son”

A

A will gives “£50,000 free of tax to my son and the residue of my estate to my wife”

Grossing-up is required where there is a specific gift to chargeable beneficiary made “free of tax” (the testator’s son) and the residue passes to an exempt beneficiary (the testator’s spouse).
The other options were incorrect because they either do not include a specific gift to chargeable beneficiary, or the specific gift is made subject to tax.

142
Q

A will contains the following gifts:

  • £20,000 (subject to tax) to my nephew
  • Residue of my estate to my civil partner

The testator has no nil rate band available when they die.

What will the testator’s nephew receive?

£20,000 less all of the inheritance tax due on the estate

£20,000 plus an amount to reflect grossing-up

£20,000

£20,000 less the proportion of inheritance tax due which is attributable to the legacy

£20,000 less half of the inheritance tax due on the estate

A

£20,000 less all of the inheritance tax due on the estate

Correct
The will leaves a specific legacy to a chargeable beneficiary ‘subject to tax’ and the residue of the estate to an exempt beneficiary. The burden of inheritance tax falls solely on the chargeable beneficiary in this situation and the benefit of the exemption applies to the whole of the exempt beneficiary’s share.
The other options were incorrect because the residue of the estate is exempt and does not bear the burden of any of the IHT. Grossing-up is irrelevant and in any event never has the effect of increasing the value of a specific legacy.

143
Q

A client comes to see you to discuss IHT planning by their will.

The client wants to make a will that leaves everything they have to their spouse (S) if the client pre-deceases S. If the client survives S, they want to leave their estate to their daughter (D).

You discuss two options:

Option 1: your client could leave everything to S.

Option 2: your client could leave a gift of the nil rate band to D and the balance of their estate to S.

Assuming your client dies before their spouse, which of the following is correct?

Whether your client chooses Option 1 or Option 2, it makes no difference to the IHT payable when your client dies.

Option 1 is preferable if your client wishes to guarantee that D will inherit a significant portion of their estate.

Whether your client chooses Option 1 or Option 2, it makes no difference to the nil rate band available when S dies.

Option 2 is preferable if S has a much smaller estate with only limited assets of their own.

Option 2 is preferable if your client would prefer the family to benefit from an increase in the nil rate band amount after your client dies.

A

Whether your client chooses Option 1 or Option 2, it makes no difference to the IHT payable when your client dies

Correct. No IHT is payable when the client dies, whether Option 1 or 2 is chosen. With Option 1, the whole estate is spouse exempt. With Option 2, the gift to D is taxed at 0% and the balance is spouse exempt. The other options are incorrect because:
It does make a difference to the NRB available for S. With Option 2, the client uses their own NRB and none will be transferred to S. With Option 1, the client’s NRB is not used and so can be transferred to S.
For the family to benefit from an increase in the NRB after the client’s death, the client should avoid using their NRB (i.e. choose Option 1). When S later claims both NRBs, the amount is calculated with reference to the NRB at the time of S’s death.
To guarantee that D will inherit, Option 2 (which includes a direct gift to D) should be chosen. If Option 1 is chosen everything falls within S’s control, and S may decide not to benefit D.
If S has a smaller estate with only limited assets and will require the use /benefit of your client’s estate to maintain an acceptable standard of living, it may not be sensible to make a significant gift to D.

144
Q

You are drafting a will for a client who wishes to make a tax-free gift to their son of the full nil rate band at the date of their death.

Your client does not want the gift to include the amount of any transferrable NRB or any residence NRB.

True or false: The following clause the best way of achieving your client’s objectives:

“I give the amount of £325,000 to my son”.

False

True

A

False

Correct. The amount of the NRB might change over time, and the client may use some of their NRB during their lifetime, so you cannot know for certain what the exact amount should be. It is preferable to use a formula clause that refers generically to the nil rate band at the date of death. However, care should be taken to ensure a formula clause is not interpreted to also include any transferred NRB or residence NRB that might be available when the client dies.

145
Q

by their will.

Your client is not married but lives with their long-term partner (P). The couple have two children together.

Your client has assets to the value of £500,000, which includes a residential property.

P has assets worth £30,000 and does not have any interest in a residential property.

Your client and P both make wills that leave the whole of their estate to each other, and on the survivor’s death their combined estates pass to their children. Assume the couple live in the property until the death of the survivor of them.

Which of the following is correct?

The residence NRB could be claimed on the death of the survivor of your client and P.

The residence NRB could only be claimed on your client’s death if your client died first.

Both NRBs will be used in full irrespective of the order of their deaths.

Both NRBs will be used in full only if P dies first.

The estate of the survivor of your client and P would be eligible for the unused proportion of the NRB of the first to die.

A

The residence NRB could be claimed on the death of the survivor of your client and P

Correct. The residence nil rate band could be claimed by the survivor of your client and P; at this point the two estates would be combined into the hands of the survivor and therefore contain the residential property interest, and the whole amount will pass to their children. The other options were incorrect because:
If P dies first, their NRB will only be used to the extent they have assets available. Their estate is well below the amount of the NRB. To the extent the NRB is unused it is wasted.
No transferrable NRB applies because your client and P are not married.
Although your client currently owns the residential interest, if they died first, the property passes to P (as the survivor), not the children and therefore the residence NRB would not be available at this time.

146
Q

A testator wishes to set up a trust under their will to benefit their children and grandchildren. The testator is uncertain what the best option is. The testator has three children and seven grandchildren.

One of the testator’s children is known to have a gambling addiction and another child is currently attending couples counselling due to marital difficulties.

All of the testator’s grandchildren are under the age of 18.

What is the most appropriate advice?

The testator should set up a life interest trust under the will appoint the children as life tenants and his grandchildren as remaindermen.

The testator should set up a life interest trust under the will and appoint the grandchildren as life tenants and his three children as remaindermen.

The testator should not leave any assets to their children by will, whether directly or into trust, because there is a risk these assets would become subject to a claim in the event of a child becoming bankrupt or getting divorced.

The testator should make an outright gift to each of the children and grandchildren to ensure that each receives a specified amount.

The testator should set up a discretionary trust under the will and name all of his children and grandchildren as beneficiaries.

A

The testator should set up a discretionary trust under the will and name all of his children and grandchildren as beneficiaries.

Correct. A discretionary trust seems most suitable. A life interest trust and outright gifts would be possible but less appropriate given the children’s circumstances and that the grandchildren are minors. A discretionary trust offers flexibility for trustees to take individual circumstances into account before distributing money (no beneficiary is entitled to anything) and a discretionary trust may offer protection from claims by creditors or on divorce.

147
Q

A woman left the whole of her estate to a two year discretionary trust under her will. Her civil partner and children are the trust beneficiaries. When she died, the woman’s estate was worth more than the nil rate band.

The 10 months after her death and after the administration of her estate was finalised the trustees appointed half of the trust fund to the woman’s civil partner and the other half to the woman’s children.

Which of the following is correct?

IHT would have been payable when the woman died. The trust distributions do not give rise to a refund of IHT because the transfers were made after her death.

No IHT was payable when the woman died as her civil partner was a beneficiary of the will trust.

No IHT was payable when the woman died as her estate was left to a discretionary trust rather than an individual person.

IHT would have been payable when the woman died. However, following the trust distributions a refund of inheritance tax can be claimed.

IHT would have been payable when the woman died. The trust distributions do not give rise to a refund of IHT because they were not made during the administration.

A

IHT would have been payable when the woman died. However, following the trust distributions a refund of inheritance tax can be claimed.

Correct. We know that IHT would have been payable following the woman’s death. Under s.144 IHTA distributions made from a discretionary will trust in the two years following death are treated as transfers made by the deceased under their will. In this case, the woman’s will is therefore treated as giving half of her estate to her civil partner and half to her children –in which case civil partner exemption will be available. A refund of some or all of the IHT paid at the time can be made.
The other options were incorrect because:
the effect of s.144 was not considered
the effect of s.144 applies in the 2 years following death and is not limited to the administration period
IHT would have been paid at the time – a discretionary trust is not an exempt beneficiary, and civil partner exemption would not have been available just because the civil partner was one of the beneficiaries

148
Q

A man was married and has three adult children from that marriage. After the man divorced his first wife he later remarried. The man has no children with his 2nd wife but she has two children of her own from her previous marriage.

The man wants to make a will is tax efficient and benefits his second wife while she is alive. He wants his wife to have a guaranteed right to his estate but open to suggestions about what form that should take. After she dies, he would like his estate to pass to his own children. The man does not want to leave any of his assets to the children of his second wife.

The man is concerned that if he gives his estate to his second wife outright she might make a will leaving everything (including the assets she inherited from him) to her own children.

The value of the man’s estate is greater than the nil rate band available.

Which of the following would you advise?

The man should create a life interest will trust naming his spouse as the life tenant and his children as the remainder beneficiaries. Spouse exemption would apply on the man’s death.

The man should create a discretionary trust and name his spouse and children as beneficiaries. Spouse exemption would apply on the man’s death.

The man should create a life interest will trust naming his spouse as the remainder beneficiary and his children as life tenants. Spouse exemption would apply on the man’s death.

The man should create a life interest will trust naming his spouse as the life tenant and his children as the remainder beneficiaries. Spouse exemption would not apply on the man’s death.

The man should create a discretionary trust and name his spouse and children as beneficiaries. Spouse exemption would not apply on the man’s death.

A

The man should create a life interest will trust naming his spouse as the life tenant and his children as the remainder beneficiaries. Spouse exemption would apply on the man’s death

Correct. Setting up a life interest will trust with his spouse as the life tenant and his children as remainder beneficiaries would achieve the clients aims. His spouse would have a guaranteed right to income and the capital of his estate would remain available for his children following her death. Spouse exemption will apply provided his spouse is named as the life tenant so this is a tax efficient structure to use in his will. The other options were incorrect because i) a discretionary trust is not suitable because his wife would have no guaranteed rights to the trust fund and ii) for spouse exemption to apply his spouse needs to be the life tenant not the remainder beneficiary.

149
Q

Can a solicitor take instructions on drafting a will from someone other than the testator?

No, a solicitor can only take instructions from the client directly.

Yes, but only if the instructions are provided by the client’s spouse or civil partner.

Yes, but only if the client provides written confirmation that the third party is authorised to act.

Yes, but only if the third party is appropriately authorised and the solicitor has no reason to suspect that the instructions do not reflect their client’s wishes.

Yes, but only if the client is present when the third party gives the instructions.

A

Yes, but only if the third party is appropriately authorised and the solicitor has no reason to suspect that the instructions do not reflect their client’s wishes.

Correct: See CCS 3.1

150
Q

What should a solicitor do if a client’s testamentary capacity is in doubt?

Consult an independent medical professional and provide them with information about the client. The solicitor should only prepare the will if, based on the advice provided by the medical professional, they believe it likely that the client has capacity.

Interview a family member of the client in order to find out more about their circumstances and make a note of findings. The solicitor should only prepare a will if they believe, following the interview, that the client has capacity. It is advisable for the family member to act as a witness.

Take the client’s instructions but make a note in the will highlighting concerns about testamentary capacity. It is advisable for the solicitor to act as a witness to the will.

Obtain the client’s consent to approach their medical practitioner for confirmation of capacity and make a record of the findings. The solicitor should only prepare a will if capacity is confirmed. It is advisable for the doctor to act as a witness and record their findings.

Refuse to act for the client and advise them to seek medical advice.

A

Obtain the client’s consent to approach their medical practitioner for confirmation of capacity and make a record of the findings. The solicitor should only prepare a will if capacity is confirmed. It is advisable for the doctor to act as a witness and record their findings

Correct: This reflects the golden rule and best practice

151
Q

Should a solicitor agree to draft a will for a client who is proposing to leave a gift to the solicitor in their will?

A solicitor is permitted to act in these circumstances but should ensure that the will is witnessed by another solicitor who can confirm the absence of undue influence.

A solicitor is prohibited from acting in these circumstances but can encourage the client to leave the property to the firm or to a member of the solicitor’s family instead.

A solicitor is absolutely prohibited from acting for a client in these circumstances.

A solicitor is not absolutely prohibited from acting in these circumstances but there is a potential conflict of interest and it is inadvisable to act.

A solicitor is not absolutely prohibited from acting in these circumstances but there is a potential conflict of interest. They should not act where the client is proposing to make a gift of more than £500 to the solicitor unless the client takes independent advice.

A

A solicitor is not absolutely prohibited from acting in these circumstances but there is a potential conflict of interest. They should not act where the client is proposing to make a gift of more than £500 to the solicitor unless the client takes independent advice.

Correct: Solicitors should not act where there is a conflict or significant risk of conflict. The SRA considers a gift of £500 to be significant.

152
Q

A testator brings you their existing valid will, on the front of which they have written “revoked”. The testator explains they wanted to revoke their existing will, but only if their son’s marriage next month goes ahead. If the wedding does not take place the testator wishes to retain the existing will.

Which of the following is correct?

The testator’s will has been revoked conditionally upon on their son getting married.

The testator’s will has not been revoked. The testator needs to wait until after their son gets married before taking steps to revoke the existing will.

The testator’s will has not been revoked. The testator should wait until after their son gets married before making a new will.

The testator’s will has been revoked and the testator should therefore revive this will or make a new will as soon as possible.

The testator’s will has not been revoked. However, it will be possible for the testator to revoke it now conditionally upon their son’s subsequent marriage.

A

The testator’s will has not been revoked. However, it will be possible for the testator to revoke it now conditionally upon their son’s subsequent marriage.

Correct: By writing “revoked” on his will the testator has not yet effectively revoked it (conditionally or otherwise). However, it would be possible for the testator to take steps now to revoke the will conditional upon his son’s subsequent marriage.
The other options were incorrect because:
- the will has not yet been revoked (conditionally or otherwise)
- it is not necessary to wait until after the marriage

153
Q

A testator made a valid will four years ago. The following year the testator executed another valid will. The second will contains the following clause: “I hereby declare this to be my last will”.

The testator died yesterday. There are no other relevant clauses within either will.

What is the effect of the second will?

The second will expressly revokes the whole of the first will.

The second will impliedly revokes the whole of the first will as it is dated later.

The second will does not expressly revoke the first and to the extent there is inconsistency the first will is given effect to.

The second will impliedly revokes the first will, but only to the extent they are inconsistent.

To be effective the second will should have been drafted to take effect only on the condition that the first will was revoked.

A

The second will impliedly revokes the first will, but only to the extent they are inconsistent

Correct. The clause in the question does not revoke the previous will. Where there are no express words of revocation both wills remain active. The later will revokes the earlier, but only to the extent they are inconsistent.
The other options were incorrect because:
- the clause included in the second will would not amount to express revocation
- it is possible for a later will to revoke an earlier will by implication
- the later will revokes the earlier where there are discrepancies (the earlier will does not take priority)
the statement relating to conditional revocation is not standard practice.

154
Q

A testator has argued with his daughter and wants to a make a new will excluding her from any benefit. The testator brings a copy of his current valid will into the office and tears it up in front of you saying “I want to revoke this will and make a new will”.

What is the effect of the testator’s actions?

The testator’s will is revoked because he carried out an act of revocation in front of a solicitor.

The testator’s will is revoked because he carried out an act of revocation with intention to revoke.

The testator’s will is revoked only to the extent his daughter benefits from it.

The testator’s will is revoked because he expressly states that he wants to make a new will.

The testator’s will remains valid.

A

The testator’s will remains valid.

Correct: Revocation by destruction requires physically destruction by the testator and intention to revoke. Although this testator intends to revoke their will revocation is only effective if the original will is destroyed. Here only a copy has been destroyed.
The other options were incorrect because:
- the original will not a copy needs to be destroyed
- there is no requirement to destroy the will in front of a solicitor
- the testator’s words do not demonstrate an intention to partially revoke
an intention to make a new will is irrelevant. An intention to revoke the existing will is required.

155
Q

A man instructs his solicitor to draw up his will. He wishes to include a gift to the solicitor of 20,000. He says this is in appreciation of the many years of excellent service he has received from his solicitor. The man estimates his estate is worth around £100,000.

Which of the following best describes the position of the solicitor?

The solicitor may draft the will but should refuse the gift as it is of significant value.

The solicitor may draft the will but should refuse the gift as he has already been paid for the work that he has carried out for the man.

The solicitor must refuse to act for the client in this situation.

The solicitor may draft the will but should refuse the gift as this is a conflict of interest under the solicitor’s code of conduct.

The solicitor may draft the will and accept the gift provided the man first takes independent legal advice.

A

The solicitor may draft the will and accept the gift provided the man first takes independent legal advice.

Correct: There is no rule which prohibits a client making a gift by will to his or her solicitor. However, paragraph 6.1 of the Code of Conduct for Solicitors requires a solicitor not to act if there is an own interest conflict or a significant risk of an own interest conflict.
If a solicitor drafts a will where the client wishes to make a gift of significant value to the solicitor, a member of their family, or an another employee of the firm, the solicitor should be satisfied that the client has first taken independent legal advice with regard to making the gift. If the client does not agree to take independent advice the solicitor should cease to act on the instructions.
This includes situations where the intended gift is of significant value in relation to the size of the client’s overall estate, but also where the gift is of significant value in itself. In practice however, most firms will have their own rules which prohibit this.

156
Q

A woman leaves the following gifts in her will:

My house to my daughter Louise

The residue of my estate to my son Theo.

Which of the following best describes where the legal burden of inheritance tax and the costs of transfer fall?

The residue bears the whole of the inheritance tax and the house will bear the cost of transfer.

The house and the residue bear the burden of inheritance tax proportionately according to their value and the house will bear the cost of transfer.

The house and the residue bear the burden of inheritance tax equally and the house will bear the cost of transfer.

The house and the residue bear the burden of inheritance tax proportionately according to their value and the residue will bear the cost of transfer.

The residue bears the whole of the inheritance tax and the costs of transfer of the house.

A

The residue bears the whole of the inheritance tax and the house will bear the cost of transfer.

Correct: In the absence of any alternative express provision in the will the burden of inheritance tax falls upon the residue of the estate and is a testamentary expense. By contrast, costs of transfer of any specific gift will fall upon the beneficiary.

157
Q

A woman dies leaving a will which provides that her property is to be held on trust for her husband for his life and after his death as such her son and daughter as survive her husband and attain the age of 25 in equal shares.

The woman’s son is 23 and her daughter is 25.

Which of the following correctly describes the respective nature of the husband, son and daughter’s interests?

The husband has a vested interest and the children have contingent interests.

The husband and daughter have contingent interests and the son has a vested interest.

The husband and daughter have vested interests and the son has a contingent interest.

The husband and children have contingent interests.

The husband and children all have vested interests.

A

The husband has a vested interest and the children have contingent interests

Correct: The husband has a vested interest in the capital. The children both have contingent interests. Whilst the daughter has satisfied the age contingency her interest is still contingent on surviving her father which is not yet satisfied. The son’s interest is contingent on reaching the age of 25 and surviving his father.

158
Q

A man leaves the following gifts in his will:

· My Rolex watch to John

· My BT shares to Althea

· £5,000 to be paid from my Sussex Building Society account to Hayley.

At the time of his death the man no longer owned the original Rolex watch but had replaced this with a new Rolex Watch, his BT holding had increased to 200 shares from the 100 shares he had when he made the will and the Sussex Building Society account had a balance of £4,000.

Which of the following best describes what, if anything, each beneficiary will receive? (You do not need to consider issues of certainty regarding the identity of the beneficiaries.)

John will receive the Rolex watch the man had when he died, Althea will receive 100 BT shares and Hayley the £4,000 in the building Society.

John will receive the Rolex watch the man had when he died, Althea will receive the 200 BT shares and Hayley the £4,000 in the building Society.

John will receive nothing. Althea will receive 100 BT shares and Hayley will receive £4,000 from the Building Society Account together with £1,000 from the general estate, provided there are sufficient funds.

John will receive nothing. Althea will receive the 200 BT shares and Hayley will the receive the £4,000 from the Building Society Account.

John will receive nothing, Althea will receive the 200 BT shares and Hayley will the receive £4,000 from the Building Society Account together with £1,000 from the general estate, provided there are sufficient funds.

A

John will receive nothing, Althea will receive the 200 BT shares and Hayley will the receive £4,000 from the Building Society Account together with £1,000 from the general estate, provided there are sufficient funds.

Correct: The watch is a specific legacy. Under s.24 Wills Act the general rule is that the will speaks from the date of death. However, use of the word ‘my’ indicates a contrary intention to the general rule indicating the man was referring specifically to the watch he owned when he made the will rather than the watch he owned at the date he died.
As regards a holding of shares, or a collection of items that is capable of growing, the use of the word ‘my’ does not alter the general position and the gift is construed as the holding or contents of the collection at the date of death.
The gift of the money in the building society is a demonstrative legacy. if there are insufficient funds in the designated account these are made up from other assets in the estate in the same way as a general legacy

159
Q

A woman leaves the residue of her estate in the following shares:

· 50% equally between my three children

· 50% equally between my brother and sister

One of the woman’s children has predeceased her leaving a daughter, the woman’s granddaughter. The woman’s brother has also predeceased her leaving a son, the woman’s nephew.

Which of the following best describes how the residue will be distributed?

The daughter’s share will pass to the granddaughter. The gift to the brother will fail and there will be a partial intestacy of his share.

The 50% share to the children will be divided between the surviving two children and the sister will take the whole of the other 50% in the residue.

The daughter’s share will pass to the granddaughter. The sister will take the whole of the other 50% share in the residue.

The daughter’s share will pass to the granddaughter and the brother’s share will pass to the nephew.

The estate will be split equally between the three surviving named beneficiaries.

A

The daughter’s share will pass to the granddaughter. The gift to the brother will fail and there will be a partial intestacy of his share

Correct: S 33 Wills Act 1837 applies so that issue (children, grandchildren, great grandchildren etc) take the share their parent would have taken. The section does not apply to any other beneficiary so not to the brother’s share. This contrasts with the statutory trusts which apply on intestacy (s 47 AEA) where the substitution provision applies to all blood relatives. It is not the case that the sister takes her brother’s share unless it is expressly stated that the gift is to such of the named beneficiaries as survive and if more than one equally. This is not implied.

160
Q

A valid will contains the following clauses:

I give £100 to my son, Michael
I £500 to my daughter, Mary
After execution, the testator made the following amendments:

Michael’s gift: The testator crossed out the ‘£100’ and inserted the replacement figure of ‘£200’.

Mary’s gift: The testator obliterated the ‘£500’, intending Mary to receive nothing. The original figure is no longer visible.

Applying s 21 Wills Act 1837, which of the following is the most accurate description of what Michael and Mary are entitled to receive?

Mary is not entitled to anything. Michael is entitled to £200.

Neither Mary nor Michael are entitled to anything.

Mary is entitled to £500. Michael is entitled to £200.

Mary is entitled to £500. Michael is entitled to £100.

Mary is not entitled to anything. Michael is entitled to £100.

A

Mary is not entitled to anything. Michael is entitled to £100.

Correct:
Applying s 21 Wills Act 1837, unattested alterations made after execution are invalid.
This means Michael cannot receive £200 (the new amount is an invalid alteration). However, as the original amount of £100 remains visible so Michael is entitled to this.
Although the alteration to Mary’s gift (the obliteration) is also unattested, as the original amount cannot be deciphered by natural means Mary receives nothing and the alteration is effective.
The other options were incorrect because:
- Mary cannot receive £500 because the amount is not visible through natural means
- Michael cannot receive £200 because the alteration has not been attested and is therefore ineffective
- Michael is entitled to receive £100 because the original clause hasn’t been amended and the amount is legible

161
Q

A testator made a valid will two years ago that contains the following clause:

“I give [£800] to my son”. The will is typed but the sum of £800 has been inserted into the blank space by hand.

The testator died yesterday and has only one son.

What is the testator’s son entitled to receive?

£800, but only if the testator inserted the sum before he executed the will.

£800, but only if the person who prepared the will signs an affidavit confirming the amendment reflected the true intention of the testator.

Nothing, as the alteration itself has not been attested.

£800, but only if the attestation clause in the will specifically refers to the alteration.

£800, as it is not necessary for the alteration to be attested.

A

£800, as it is not necessary for the alteration to be attested.

Where a blank space has been completed (inserting £800) there is a presumption that the blank was completed before execution. In which case the alteration would be valid under the general rule in s 21 Wills Act 1837.
The other options were incorrect because:
- There is a presumption the blank space was completed before execution so this does not have to be proved in order for the gift to be valid
- Attestation of the alteration is not required where a blank space is completed
- The attestation of the will as a whole does not need to refer to the alteration (completing a blank space is deemed to have occurred before execution)
Affidavit evidence is not required

162
Q

A testator made a valid will two years ago that contains the following clause:

“I give £100 to my nephew”

The ‘£100’ has been crossed out by hand but remains visible underneath.The attestation clause in the will makes no reference to the amendment. The testator died yesterday and has only one nephew. You do not know when the testator made the alteration to his will.

What is the testator’s nephew entitled to receive?

£100, but only if the person who prepared the will signs an affidavit confirming the alteration was made before execution.

£100, but only if the witnesses to the will sign an affidavit confirming the alteration was made before execution.

Nothing, on the face of it the amendment demonstrates a clear intention to revoke the gift.

Nothing, under the general rule in s 21 Wills Act 1837.

£100, under the general rule in s 21 Wills Act 1837.

A

£100, under the general rule in s 21 Wills Act 1837.

Correct: The general rule in s.21 Wills Act 1837 is that unattested alterations are deemed to have been made after execution and are therefore invalid. In the absence of evidence to the contrary, the alteration is ineffective. The original gift is visible through natural means and so the nephew will inherit this.
The other options were incorrect because:
- There is a presumption the alteration was made after execution, so no affidavit evidence is required either by witnesses or the person who prepared the will.
- The general rule does not confirm the validity of the amendment
Crossing a line through a number does not of itself demonstrate intention to revoke

163
Q

A testator made a valid will five years ago which includes the following clause:

“3. I give £400 to my youngest daughter”

The testator made a valid codicil to this will six months ago. The codicil did not expressly refer to or amend clause 3 of the will. When the will was executed the testator had two daughters. When the codicil was executed, the testator had just given birth to her third daughter.

The testator died last month. What is the effect of clause 3 of the will?

The testator’s second daughter will receive £400 because she was the youngest daughter when the will was made.

Clause 3 does not take effect because there is ambiguity regarding which of the daughters it refers to.

The testator’s second daughter will receive £400 because she was the youngest daughter when the will was made and the codicil did not amend this clause of the will.

The testator’s third daughter will receive £400 because she was the youngest daughter when the codicil was made.

The testator’s second daughter will receive £400 because it is clear this must have been the intention of the testator.

A

The testator’s third daughter will receive £400 because she was the youngest daughter when the codicil was made.

Correct: The codicil re-publishes the will which is interpreted with reference to the date of the codicil. On the date the codicil was executed the testator had three daughters and the youngest of them will receive £400. The other options were incorrect because:
- The effective date of execution of the will is the date of the codicil
- The codicil republishes the will as a whole not just the clauses it amends
There is no reference in the codicil to how the clause should take effect the general legal principles apply

164
Q

A testator made a valid will two years ago. The will was witnessed by his son and his neighbour and included the following clauses:

“3. I give £30,000 to my son

  1. I give £30,000 to my daughter”

The testator made a valid codicil to his will a few months later which was witnessed by his two neighbours.

After executing the will, but before executing the codicil, the testator made a manuscript amendment to clause 4 of his will by drawing a line through the amount of £30,000 and replacing it with £50,000. No express reference to the amendment to the will was made in the codicil. The original amount of £30,000 remains visible.

What will the testator’s son and daughter receive?

The testator’s son will receive nothing as he witnessed the will. The testator’s daughter will receive £50,000, but only if evidence that the amendment was made before the codicil was executed is provided.

The testator’s son will receive nothing as he witnessed the will. The testator’s daughter will receive £50,000. No additional evidence proving when the amendment was made is needed because the date of the codicil is after the date of the will.

The testator’s son will receive £30,000. The testator’s daughter will receive nothing.

The testator’s son will receive £30,000. The testator’s daughter will receive £50,000, but only if evidence that the amendment was made before the codicil was executed is provided.

The testator’s son will receive nothing as he witnessed the will. The testator’s daughter is only entitled to £30,000.

A

The testator’s son will receive £30,000. The testator’s daughter will receive £50,000, but only if evidence that the amendment was made before the codicil was executed is provided.

Correct: The testator’s son witnessed the will. According to s.15 Wills Act 1837 he would not be entitled to the gift. However, the later codicil, which he did not witness, re-executes the will and therefore avoids the effect of s.15.
The amendment to the daughter’s gift was made after the will was executed so would be ineffective. However, the later codicil re-executes the will (by which time it includes the amended clause) and so the amendment is confirmed. As the codicil does not expressly confirm the amendment was made before the codicil was signed affidavit evidence to this effect would be required.
The other options were incorrect because the:
- effect of s.15 is avoided - the codicil was not witnessed by the son and this ‘corrects’ the issue.
- testator’s daughter will receive something – either the original amount or the increased amount if the alteration is proven to be effective
- date of the codicil is, of itself, insufficient to validate the amendment. Evidence is needed to prove the amendment was made before the codicil was signed as the codicil itself does not expressly refer to the amendment to the will.

165
Q

You have received three sets of instructions from the following testators:

A testator who revoked their will by destroying it. Testator 1 regrets their actions and wants to revive the will again and include some additional clauses.
A testator who made a will two years ago and subsequently married has just found out that by getting married his will was automatically revoked. Testator 2 wants to revive their previous will and include some additional clauses.
A testator who made a will last year now wishes to amend it. Testator 3 wants to revoke part of the will. There are no additional clauses to add.

Which of the testators could achieve their aim by making a codicil?

Testator 2 and 3 only.

Testator 2 only.

Testator 1 and 2 only.

Testators 1, 2 and 3.

Testator 1 and 3 only.

A

Testator 2 and 3 only.

A codicil can revive a will that was previously revoked, but not if it has been destroyed. This means Testator 1 could not use a codicil to revive his previous will but Testator 2 could.
A codicil can be used to revoke a will in whole or in part. Testator 3 can therefore use a codicil. It does not matter that there are no additional clauses to add.

166
Q

A testator died last week. The testator made her first valid will three years ago.

The will appoints the testator’s husband to be joint executor of her estate with her brother. The will contains one gift of £300 to her nephew and the rest of her assets are shared equally between such of her children and husband that survive her.

The testator and her husband finalised their divorce last year. The testator’s husband is still alive.

Which of the following best describes the effect of the will?

The will was partially revoked when the testator divorced. The testator’s former spouse and brother will be appointed executors, the testator’s nephew will inherit £300 and the rest of her estate is shared between her children only.

The will is not affected by the divorce as it was executed beforehand. The clauses take effect as stated.

The will was automatically revoked in full when the testator divorced. The testator died intestate.

The will was partially revoked when the testator divorced. The testator’s brother will act as sole executor, the testator’s nephew will inherit £300 and the rest of her estate is shared between her former spouse and children.

The will was partially revoked when the testator divorced. The testator’s brother will act as sole executor, the testator’s nephew will inherit £300 and the rest of her estate passes to her children only.

A

The will was partially revoked when the testator divorced. The testator’s brother will act as sole executor, the testator’s nephew will inherit £300 and the rest of her estate passes to her children only.

Correct: By virtue of s 18A Wills Act 1837 (‘WA’) the will is interpreted as though the testator’s former spouse has pre-deceased her (a partial revocation). This means only the brother can act as executor and the former spouse cannot take a share of her estate; the rest of her assets (after the gift to her nephew is paid) will be shared between her children only.
The other options were incorrect because:
- Divorce does not fully revoke the will – the gift to the testator’s nephew would be unaffected
- S 18A WA prevents the appointment of the former spouse as executor and trustee as well as rendering their beneficial entitlements void
- S 18A WA affects all wills/codicils made prior to the divorce

167
Q

A testator made a valid will four years ago which contained the following clause:

“This will is made in contemplation of any civil partnership I subsequently enter”.

The testator entered into a civil partnership one year later. Two months ago the testator separated from their civil partner and started proceedings to dissolve the civil partnership.

The testator died yesterday before any court order was finalised. There are no other relevant clauses in the will.

Which of the following is correct in respect of the testator’s will?

The will was automatically revoked when the testator entered the civil partnership. If the testator has made a previous will, their estate will be distributed according to the terms of the previous will.

The will was automatically revoked when the testator separated from their civil partner.

The will was automatically revoked when the testator entered the civil partnership and the testator died intestate.

The will was partially revoked when the testator initiated formal proceedings to dissolve the civil partnership.

The testator’s will has not been revoked as no court order confirming the dissolution of the civil partnership was issued before they died.

A

The will was automatically revoked when the testator entered the civil partnership and the testator died intestate.

Correct: By virtue of s 18 B Wills Act 1837 (‘WA 1837’) entering into a civil partnership automatically revokes any previous will made by the testator. The express clause in the testator’s will concerns a hypothetic civil partnership and does not identify a named civil partner – it is therefore ineffective.
The other options were incorrect because:
S 18C WA 1837 does not take effect until a court order confirming dissolution has been issued
S 18B WA 1837- entering a civil partnership revokes all previous wills in full, not just the last one

168
Q

A testator plans to get married next month. The testator made a valid will (Will 1) years before meeting their fiancé (F) and leaving all of their assets to their sister.

The testator now wants to make a new will (Will 2) leaving most of their assets to their future spouse.

If the testator executes Will 2 before the wedding, which of the following is correct?

Will 2 will not be revoked by the testator’s marriage, provided it is made expressly in contemplation of the testator’s marriage. No reference to F is required.

Will 2 will not be revoked by the testator’s marriage, provided it is made expressly in contemplation of the testator’s marriage to F.

Will 1 is unaffected by the testator’s marriage.

Will 2 should not have been executed before the wedding and can only remain effective if the wedding does not take place.

Will 2 will not take effect until the testator’s marriage, provided it is made in contemplation of the testator’s marriage to F.

A

Will 2 will not be revoked by the testator’s marriage, provided it is made expressly in contemplation of the testator’s marriage to F.

Correct: By virtue of s 18 Wills Act 1837 marriage automatically revokes any will or codicil made beforehand. This can only be avoided if the will includes an express clause stating it is made in contemplation of the marriage to F and should not be revoked by it.
The other options were incorrect because:
- Whether or not a will is made in contemplation of marriage, the will is effective on execution unless there is express wording that it should only take effect on the occurrence of particular event
- Will 1 is revoked by the marriage.
Will 2 can be made before the wedding provided it is made expressly in contemplation of that wedding.